Neck Qs

Ace your homework & exams now with Quizwiz!

77. Cerebrospinal fluid is formed by vascular choroid plexus in which structure?

A Cerebrospinal fluid is formed by vascular choroid plexus in the ventricles in the brain; the letter "A" indicates the lateral ventricle.

84. Which structure is formed by the perpendicular plate of the ethmoid bone, vomer, and septal cartilage?

A The nasal septum is formed primarily by the perpendicular plate of the ethmoid bone, vomer, and septal cartilage.

53. Lesion of the trunks of the brachial plexus is most likely to occur from a penetrating wound into which of the following labeled areas in the given drawing of the neck regions?

A The neck can be demarcated into anterior and lateral cervical regions by the obliquely running sternocleidomastoid (SCM) muscle. Both of these regions can be further subdivided into smaller triangular areas relative to the SCM and other neck muscles. This schematic plan allows a topographic organization of the structures within the neck. The lateral cervical region (or posterior cervical) triangle is demarcated by the posterior border of the SCM, anterior border of the trapezius, and the middle part of the clavicle. It is subdivided into an upper, larger occipital triangle and a lower, smaller omoclavicular (subclavian or supraclavicular) triangle by the course of the inferior belly of the omohyoid muscle. The roots of the brachial plexus emerge from the interscalene triangle (scalene hiatus) to form the three trunks of the brachial plexus in the occipital triangle. These trunks descend through the occipital triangle and form the divisions of the plexus as they approach the clavicle. Penetrating wounds into the occipital triangle are highly likely to contact some part of the trunks of the brachial plexus. Choice B (Omoclavicular triangle) is incorrect. The omoclavicular (subclavian or supraclavicular) triangle is the lower, smaller part of the lateral cervical region (or posterior cervical triangle). Its major contents include the third part of the subclavian artery, the suprascapular artery, and supraclavicular lymph nodes. A deep penetrating wound here may damage the divisions of the brachial plexus. Choice C (Submandibular triangle) is incorrect. The anterior cervical region (or triangle) is demarcated by the anterior border of the SCM, inferior border of the mandible, and the anterior midline of the neck. It is subdivided into four regions (or triangles) by the courses of the superior belly of the omohyoid and both bellies of the digastric muscles: the submandibular, submental, carotid, and muscular triangles. The submandibular (digastric) triangle is bounded by the mandible and the two bellies of the digastric muscle. Its major contents include the submandibular salivary gland, hypoglossal nerve (CN XII), and portions of the facial artery and vein. Trauma with this triangle would not affect the brachial plexus. Choice D (Carotid triangle) is incorrect. The carotid triangle represents a crowded region that is bounded by the SCM, superior belly of the omohyoid, and posterior belly of the digastric muscles. It houses several major neurovascular structures, including the common carotid artery and its branches, internal jugular vein, vagus and hypoglossal nerves, ansa cervicalis, and deep cervical lymph nodes. Trauma with this triangle would not affect the brachial plexus. Choice E (Muscular triangle) is incorrect. The muscular triangle is a large area demarcated by the SCM, superior belly of the omohyoid, and the anterior midline of the neck. It is occupied mainly by the infrahyoid (strap) muscles, and also includes the larynx and the thyroid and parathyroid glands. Trauma with this triangle would not affect the brachial plexus.

"27. A 42-year-old man has a lymph node biopsy in the left side of his lateral cervical region or posterior triangle of his neck. After closure of the wound, the physician asks the patient to rotate his head to the right against resistance, as shown in the photo. What nerve is the doctor assessing with this test? (A) Hypoglossal (B) Accessory (C) Mandibular division of trigeminal (D) Greatauricular (E) Upper trunk of the brachial plexus"

Accessory The accessory nerve (CN XI) passes deep to the sternocleidomastoid muscle and to the investing layer of deep cervical fascia and courses posterolaterally into the lateral cervical region (posterior triangle of the neck). Due to its superficial course, it is at risk during a lymph node biopsy, cannulation of the internal jugular vein, carotid endartectomy, stab wounds, or removal of a melanoma in the lateral cervical region. CN XI innervates only the sternocleidomastoid muscle and the trapezius muscle, which are primarily involved in head and shoulder movements. Specifically, the left sternocleidomastoid muscle contracts to bring the mastoid process of the temporal bone closer to the sternum, which results in tilting the head toward the left side and elevation of the chin to the right. The elevation of the chin to the right is the action being tested by this physician, so the accessory nerve is the correct answer. Choice A (Hypoglossal nerve) is incorrect. The left hypoglossal nerve (CN XII) is located in the anterior cervical region, so it would not be damaged by a lymph node biopsy in the lateral cervical region, or posterior triangle of the neck. Moreover, CN XII innervates all of the intrinsic muscles of the tongue and most of its extrinsic muscles with the lone exception being the palatoglossus muscle, innervated by the vagus nerve (CN X). Therefore, damage to the left CN XII would cause ipsilateral deviation of the tongue during protrusion and cause dysarthria, or difficulty speaking. In this patient, the doctor is testing the rotation of the head to the right, so the hypoglossal nerve is not being assessed. Choice C (Mandibular division of trigeminal) is incorrect. The mandibular (third) division of the trigeminal nerve (CN V3) is the only division of the trigeminal nerve (CN V) that supplies motor innervation. It supplies the muscles derived from the mesoderm of the first pharyngeal arch, including the four muscles of mastication (temporalis, masseter, lateral pterygoid, and medial pterygoid) and four additional muscles: Mylohoid, Anterior belly of the Digastric, Tensor Tympani, and Tensor Veli Palatini (mnemonic = "MATT"). These muscles play no role in rotation of the head, and CN V3 is not located in the lateral cervical triangle where the biopsy occurred. Choice D (Great auricular) is incorrect. The great auricular nerve does course through the lateral cervical region; however, it is a cutaneous (sensory) nerve supplying the skin over the mastoid process of the temporal bone, parotid gland, and auricle. It does not provide any motor innervation, so this nerve would not be involved with rotation of the head to the right side. Choice E (Upper trunk of the brachial plexus) is incorrect. The upper trunk of the brachial plexus is comprised of the anterior (ventral) rami of C5 and C6, and it emerges between the anterior and middle scalene muscles in the lateral cervical region (posterior triangle of neck). However, damage to the upper trunk of the brachial plexus would result in an abnormal postural presentation of the upper limb known as the "waiter's tip" deformity (or Erb-Duchenne palsy). The upper trunk of the brachial plexus is not involved with rotation of the head to the right side.

"41. A tumor growing at the base of the skull impinges upon the opening indicated by the arrow, severely compressing its contents. Which of the following conditions is the most likely result? (A) Venous drainage from the base of the brain is obstructed (B) Mucus secretion in the oral floor is reduced (C) Sensation from the mandibular teeth is lost (D) Motor control of the upper pharynx is lost (E) Arterial supply to the dura mater is reduced"

Arterial supply to the dura mater is reduced The indicated opening is the foramen spinosum. This small foramen conveys the middle meningeal artery (a branch of the maxillary artery) and the spinous nerve (a branch of the mandibular division of the trigeminal nerve, or CN V3) from the infratemporal fossa into the cranium, in the floor of the middle cranial fossa. The middle meningeal artery provides the major blood supply to the dura mater and the cranial bones. It does not supply the brain. The middle meningeal artery is often involved with epidural hematomas. Choice A (Venous drainage from the base of the brain is obstructed) is incorrect. The main venous drainage of the brain is through the dural venous sinuses into the internal jugular veins, which exit the cranial cavity through the jugular foramina. This primary drainage route is supplemented by several emissary veins that pass through various other openings in the skull, including the foramen ovale and carotid canal in the floor of the middle cranial fossa. However, the foramen spinosum does not normally carry emissary veins. Choice B (Mucus secretion in the oral floor is reduced) is incorrect. Mucus secretion in the oral floor is controlled by parasympathetic neurons within the chorda tympani nerve. The chorda tympani branches from the facial nerve in the facial canal, passes through the tympanic (middle ear) cavity, and leaves that space through a small opening (petrotympanic fissure) to enter the infratemporal fossa. Choice C (Sensation from the mandibular teeth is lost) is incorrect. Afferent neurons from the mandibular teeth are carried in the inferior alveolar nerve, a branch of the mandibular division of the trigeminal nerve (CN V3). CN V3 passes through the foramen ovale in the base of the skull. The inferior alveolar nerve passes through the mandibular foramen as it leaves the mandible conveying sensation from the mandibular teeth. Choice D (Motor control of the pharyngeal constrictor muscles is lost) is incorrect. Motor control of the pharyngeal constrictor muscles is from branches of the vagus nerve (CN X). The vagus exits the cranial cavity through the jugular foramen, in company with the glossopharyngeal (CN IX) and accessory (CN XI) nerves.

74. A middle ear infection may spread into which structure?

B Mastoid air cells communicate with the middle ear cavity through the antrum and aditus.

55. Which of the labeled structures is derived from the body of the first cervical vertebra in this labeled X-ray showing a transoral view of the upper cervical vertebrae?

B Odontoid process. The image is an anteroposterior (AP) view of the upper cervical spine through the open mouth. This transoral radiological imaging view is valuable for evaluation of this region, especially for relations of the C1 and C2 vertebrae. The odontoid process (or dens) of C2 is a distinctive, tooth-like projection situated between the lateral masses of C1. It articulates with the anterior arch of C1, with the resultant middle atlantoaxial joint producing the major degrees of rotation within the cervical spine. Developmentally, the odontoid process begins as the body of the C1 vertebra. It separates from C1, migrates caudally, and joins the body of C2 to form the odontoid process. Thus, C1 is left with no body and possesses only the small anterior arch at that origin site. Choice A (Base of skull) is incorrect. The skull is divided into two main parts: (1) the neurocranium, forming the brain case; (2) the viscerocranium, forming the facial skeleton. The neurocranium is subdivided into two parts based on topographic and developmental features: (1) the chondrocranium (cartilaginous neurocranium), formed by endochondral ossification and composing the base of the skull; (2) the membranous neurocranium, formed by membranous ossification and composing the flat bones of the cranial vault. The base of the skull indicated in the X-ray is the occipital region, a part of the chondrocranium. It develops from occipital sclerotomes derived from the paraxial mesoderm. Choice C (Inferior articular facet of C1) is incorrect. The inferior articular facet (process) of C1 develops as an outgrowth from the vertebral (neural) arch of its incipient vertebral element. Choice D (Spinous process of C2) is incorrect. The spinous process of C2 develops as an outgrowth from the vertebral (neural) arch of its incipient vertebral element. Choice E (Transverse process of C1) is incorrect. In the cervical region, the transverse process of C1 is formed by contributions from both the vertebral arch and costal process elements.

15. A 27-year-old paratrooper lands on a pine tree. Consequently, preganglionic parasympathetic nerves leaving the central nervous system are lacerated. Which of the following structures contain cell bodies of the damaged nerve fibers? (A) Cervical and sacral spinal cord (B) Cervical and thoracic spinal cord (C) Brain stem and cervical spinal cord (D) Thoracic and lumbar spinal cord (E) Brain stem and sacral spinal cord

Brain stem and sacral spinal cord Preganglionic neurons of the parasympathetic nervous system are located in the brain stem (cranial outflow) and sacral spinal cord segments S2-S4 (sacral outflow). Preganglionic sympathetic neurons are located in the thoracic and lumbar spinal cord.

"29. A 10-year-old girl presents with a smooth, round neck mass the size of a golf ball at the upper third of the anterior border of her right sternocleidomastoid muscle, as seen in the photo. Her mother says this once small, peanut-sized mass grew without pain or inflammation after it became noticeable 2 months previously. The mass does not affect the girl's daily activities. What is the most likely diagnosis for the pictured mass? (A) Thyroglossal cyst (B) Branchial cyst (C) Undescended thymus (D) Preauricular cyst (E) Ectopic palatine tonsil"

Branchial cyst The second pharyngeal (branchial) arch normally overgrows the more caudal pharyngeal arches and merges with the epicardial ridge in the lower part of the developing neck region. This development causes the second, third, and fourth pharyngeal clefts (grooves) to lose contact with the surface of the neck and coalesce into a common cervical sinus. The cervical sinus is usually obliterated early in development. However, if the second arch does not properly grow caudally, remnants of the second, third, and/or fourth clefts may persist as a branchial cyst (lateral cervical cyst; cervical lymphoepithelial cyst) along the lateral side of the neck. This type of cyst may or may not be connected to the surface by a small drainage canal (branchial fistula). Thus, branchial cysts are the remnants of the cervical sinus and its duct. They are located along the anterior border of the SCM. Most often, these cysts are the remnants of the second pharyngeal cleft, located just below the angle of the mandible. Second branchial cleft cysts represent approximately 67% to 93% of all pharyngeal apparatus anomalies. However, branchial cysts may be found anywhere along the anterior margin of the SCM. Very frequently, branchial cysts are inconspicuous at birth, becoming evident as they enlarge throughout childhood. Choice A (Thyroglossal cyst) is incorrect. The thyroid gland first appears as a single, ventromedian diverticulum (thyroid diverticulum) off the floor of the embryonic pharynx, between the tuberculum impar and copula of the incipient tongue. It descends along the midline, anterior to the gut tube, remaining connected to the tongue by a narrow canal (the thyroglossal duct). The thyroglossal duct usually solidifies and is obliterated after the final descent of the thyroid gland into its normal terminal position. A thyroglossal cyst is a cystic remnant of the thyroglossal duct. It is always located in or close to the midline of the neck. Most commonly (∼50%), it is found near the body of the hyoid bone. However, a thyroglossal cyst may lie anywhere along the normal migratory route of the thyroid gland. Choice C (Undescended thymus) is incorrect. The thymus gland begins as epithelial primordia in the ventral wings of both the paired third pharyngeal (branchial) pouches. Losing their connections with the pharyngeal walls, these epithelial primordia descend caudally and medially and ultimately migrate into the upper anterior thorax where they fuse with each other. Sometimes, the tail portion of one or both of the descending primordia may persist in the neck. In such cases, thymic tissue may be embedded in the thyroid gland or may be found as isolated thymic pockets near the midline. Partial or complete absence of the thymus gland is a third pharyngeal pouch component of DiGeorge syndrome, a complex collection of craniofacial and cardiovascular anomalies. Choice D (Preauricular cyst) is incorrect. The auricle (pinna) of the external ear develops from multiple proliferations (auricular hillocks) in the dorsal ends of the first and second pharyngeal (branchial) arches, surrounding the opening of the first pharyngeal cleft. Its initial position is in the incipient neck region, with development of the neck and mandible causing the auricle to ascend to its final position at the side of the head. Development of the auricle is complicated, and congenital abnormalities of the auricle (preauricular cysts and pits, auricular sinuses, skin tags) are common. Importantly, auricular malformations are often associated with other congenital disorders. Choice E (Ectopic palatine tonsil) is incorrect. The fossa, epithelium, and crypts of the palatine tonsils are derived from the second pharyngeal (branchial) pouches and are related to the developing oropharyngeal region. The lymphoid tissue of the tonsil is a secondary infiltration of that bed. Ectopic lymphoid tissue may be found in small nodules near the tonsillar fossa.

73. Which structure lies lateral to the lateral wall of the nasal cavity and inferior to the floor of the orbit?

C The maxillary sinus lies lateral to the lateral wall of the nasal cavity and inferior to the floor of the orbit.

70. A 46-year-old man visits the speech therapist complaining of dryness of the mouth. The therapist performs a swallowing study and, on examination, finds that the man has a lack of salivary secretion from the submandibular gland. This indicates a lesion of which of the following nervous structures? (A) Lingual nerve at its origin (B) Chorda tympani in the middle ear cavity (C) Superior cervical ganglion (D) Lesser petrosal nerve (E) Auriculotemporal nerve

Chorda tympani in the middle ear cavity . The chorda tympani nerve contains preganglionic parasympathetic fib- ers responsible for secretion of the submandibular gland. The lingual nerve at its origin is not yet joined by the chorda tympani. The superior cervical ganglion provides sympathetic fibers, which supply blood vessels in the submandibular gland. The lesser petrosal nerve contains preganglionic parasympathetic fibers that synapse in the otic ganglion. The auriculotemporal nerve contains postganglionic parasympathetic fibers, which are responsible for secretion of the parotid gland.

26. A 41-year-old woman overdoses on some prescription medications that have a common side effect of autonomic nerve stimulation. Which of the following conditions or actions results from stimulation of the parasympathetic fibers to the eyeball? (A) Enhanced vision for distant objects (B) Dilation of the pupil (C) Contraction of capillaries in the iris (D) Contraction of the ciliary muscle (E) Flattening of the lens

Contraction of the ciliary muscle When the parasympathetic fibers to the eyeball are stimulated, the pupil constricts and the ciliary muscle contracts, resulting in a thicker lens and enhanced vision for near objects (accommodation). Dilation of the pupil, contraction of capillaries in the iris, and enhanced ability to see distant objects (flattening of the lens) result from stimula- tion of sympathetic nerves.

13. A baby girl presents with a disproportionately wide skull with a short occipitofrontal diameter, as depicted in the given surface shaded CT reconstruction. What cranial suture most likely closed prematurely to result in this cranial deformity? (A) Sphenosquamous (B) Sphenoparietal (C) Lambdoid (D) Coronal (E) Sagittal

Coronal Premature closure of the coronal suture leads to brachycephaly (G: short head), which leads to a disproportionately wide skull with a short occipitofrontal diameter. The surface shaded CT reconstruction shows the complete closure of the coronal suture and depicts the characteristic square-shaped skull with a short occipitofrontal diameter seen following premature closure of the coronal suture. Brachycephaly is more common in females, and surgical intervention can be implemented to remove bone from both coronal sutures. Interestingly, some infants wear molding caps to treat cranial deformities, if surgical intervention is not required. If only one side of the coronal suture closes prematurely, the infant would present with an asymmetric cranium, a condition known as plagiocephaly. Craniosynostosis is the term that refers generally to the premature fusion of the cranial sutures. Choice A (Sphenosquamous) is incorrect. The sphenosquamous suture is a dense, fibrous connective tissue joint located on the side of the skull between the greater wing of the sphenoid bone and the squamous portion of the temporal bone. This suture later closes to help form the pterion, which is clinically relevant due to the fractures located at this structurally weak area on the side of the head, which may cause epidural hemorrhage via damage to the middle meningeal artery. The premature closure of this suture, as well as the sagittal, parietotemporal, and sphenoparietal sutures, can be involved with scaphocephaly (G: boat-shaped skull). Choice B (Sphenoparietal) is incorrect. The sphenoparietal suture is a dense, fibrous connective tissue joint located on the side of the skull between the greater wing of the sphenoid bone and the parietal bone. This suture later closes to help form the pterion, which is clinically relevant due to the fractures located at this structurally weak area on the side of the head, which may cause epidural hemorrhage via damage to the middle meningeal artery. The premature closure of this suture, as well as the sagittal, parietotemporal, and sphenosquamous sutures, can be involved with scaphocephaly (G: boat-shaped skull). Choice C (Lambdoid) is incorrect. The lambdoid suture is a dense, fibrous connective tissue joint located on the back of the skull that connects the occipital bone with the posterior aspect of the parietal bone and petrous portion of the temporal bone. If only one side of the lambdoid suture closes prematurely, the infant would present with a twisted and asymmetric cranium, a condition known as plagiocephaly. Choice E (Sagittal) is incorrect. The sagittal suture is a dense, fibrous connective tissue joint located between the two parietal bones in the midline of the skull. The premature closure of this suture, as well as the parietotemporal, sphenosquamous, and sphenoparietal sutures, can be involved with scaphocephaly (G: boat-shaped skull), which presents as a long and narrow cranium.

4. During surgery on a 56-year-old man for a squamous cell carcinoma of the neck, a surgeon notices profuse bleeding from the deep cervical artery. Which of the following arteries must be ligated immediately to stop bleeding? (A) Inferior thyroid artery (B) Transverse cervical artery (C) Thyrocervical trunk (D) Costocervical trunk (E) Ascending cervical artery

Costocervical trunk The surgeon should ligate the costocervical trunk because it divides into the deep cervical and superior intercostal arteries. The thyrocervical trunk gives off the suprascapular, transverse cervical, and inferior thyroid artery. The ascending cervical artery is a branch of the inferior thyroid artery.

A 68-year-old man was choking on a piece of steak at a family restaurant. Despite attempts to dislodge the food via abdomi- nal thrusts (or the Heimlich maneuver), his upper airway remained blocked. An emergency medical technician (EMT), eating at the scene, performed an emergency tracheotomy to enable the man to breathe. Which subcutaneous structure was most likely cut during this procedure? (A) Cricoidcartilage (B) Thyrohyoid membrane (C) Cricothyroid membrane (D) Trachealrings (E) Isthmus of thyroid gland

Cricothyroid membrane In an emergency tracheotomy, the cricothyroid membrane is incised in order to establish a direct airway for the patient. This procedure is also called a cricothyrotomy or cricothyroidotomy, and it is used as a last resort to circumvent upper airway obstructions. The cricothyroid membrane is an important component of the conus elasticus, which is composed of the vocal ligaments, median cricothyroid membrane, and lateral cricothyroid membranes. The cricothyroid membrane is the perfect location to perform an emergency tracheotomy because of several nearby palpable landmarks, and it is located below the (true) vocal folds, which serve as the main inspiratory sphincter of the larynx. Do not confuse the emergency tracheotomy with a tracheostomy, which is a procedure performed in a hospital setting and involves surgically creating a hole in the cartilaginous rings of the trachea. Choice A (Cricoid cartilage) is incorrect. Due to the thickness of the cricoid cartilage, incising through this cartilage would be difficult outside the hospital setting. Moreover, an incised cricoid cartilage would need surgical intervention to heal due to its lack of blood supply. Moreover, damaging the cricoid cartilage would be detrimental to the integrity of the larynx and the laryngeal skeleton. Choice B (Thyrohyoid membrane) is incorrect. Though it is easily palpated due to its position above the laryngeal prominence (or the "Adam's apple"), the thyrohyoid membrane is located between vertebral levels C3 and C4 and may not establish a direct airway. To combat against an upper airway obstruction, the cricothyroid membrane (between vertebral levels C5 and C6) would be a better option. Damage to the thyrohyoid membrane could also compromise the superior laryngeal artery and the internal branch of the superior laryngeal nerve, which pierce this membrane to enter the larynx. Choice D (Tracheal rings) is incorrect. In a tracheostomy, a hole is created surgically in the cartilaginous rings of the trachea. However, this procedure is usually performed in a hospital setting under sterile conditions. Damage to the thyroid gland and infrahyoid muscles can easily occur if a tracheostomy is not performed correctly. Incising the cricothyroid membrane would be a much easier means of establishing an airway, especially considering the emergency conditions surrounding this patient's choking incident. Choice E (Isthmus of the thyroid gland) is incorrect. Cutting through the isthmus of the thyroid gland would not establish an airway for this patient, so this option can be easily eliminated. Due to its location at the seventh cervical vertebra, this glandular tissue is often transected or resected during a tracheostomy, when the tracheal rings are incised. However, a tracheostomy is performed in a hospital setting under sterile conditions.

75. Which structure has numerous small cavities and lies between the orbit and the nasal cavity?

D The ethmoid sinus has numerous small cavities and lies between the orbit and the nasal cavity.

45. Which of the following labeled areas indicates the petrous portion of the temporal bone?

D Petrous portion of temporal bone. The petrous portion of the temporal bone is a large wedge-shaped structure extending from the side of the skull toward the midline, forming much of the boundary between the middle and posterior cranial fossae. It is the densest bone in the body, being composed almost entirely of compact bone tissue. As a result, it forms a strong radiopaque image in plain films and CT scans. The image here is a PA (Posterior-Anterior) plain film (X-ray) of the head. The petrous part of the temporal bone appears as a distinct opaque structure with a sharp upper margin (petrous ridge) across the lower aspect of the orbit. Thus, it serves as a notable orientation structure in this view. Choice A (Superior orbital rim) is incorrect. The upper rim of the orbit is part of the frontal bone. It forms a distinct radiopaque curve well above the petrous ridge in the PA view. Choice B (Squamous part of temporal bone) is incorrect. The squamous portion of the temporal bone is a relatively thin, flat plate on the side of the skull. The curvature of the skull, plus the angle of this view, accounts for the opaque edge seen here. Choice C (Mastoid process) is incorrect. The mastoid process is a breast-like appendage of the temporal bone, housing the extensive mastoid air cells (mastoid sinuses). This pneumatic composition results in the mastoid process appearing as a ghost-like, nearly radiolucent image in the PA view. Choice E (Upper alveolar margin) is incorrect. The upper alveolar margin is the tooth-bearing part of the maxillary bone. It forms a horseshoe-shaped arch under the nasal cavity in this view. The appearance of the arcade may be intensified by the presence of dental fillings.

20. The nerve accompanying the superior thyroid artery may be damaged during an operation on the thyroid gland. Which of the following functional defects may result from this injury? (A) Loss of sensation above the vocal cord (B) Loss of lateral rotation of the arytenoid cartilages (C) Paralysis of the vocalis muscle (D) Lack of abduction of the vocal cord (E) Decreased tension of the vocal cord

Decreased tension of the vocal cord he superior thyroid artery is accompanied by the external laryngeal nerve, which innervates the cricothyroid muscle. Paralysis of this muscle due to a lesion of the external laryngeal nerve decreases tension of the vocal cord. Loss of sensation above the vocal cord is due to injury of the internal laryngeal nerve. The posterior cricoarytenoid muscle draws the muscular process of the arytenoid cartilage posteriorly and thereby rotates its vocal process laterally. Paralysis of the vocalis muscle is due to a lesion of the recurrent laryngeal nerve. Lack of abduction of the vocal cord results from paralysis of the posterior cricoarytenoid muscle.

54. An angiogram of a 45-year-old man shows an occlusion of the costocervical trunk. This obstruction could produce a marked decrease in the blood flow in which of the following arteries? (A) Superior thoracic artery (B) Transverse cervical artery (C) Ascending cervical artery (D) Deep cervical artery (E) Inferior thyroid artery

Deep cervical artery The costocervical trunk gives rise to the deep cervical and superior inter- costal arteries. The superior thoracic artery arises from the axillary artery. The transverse cervical, inferior thyroid, and suprascapular arteries arise from the thyrocervical trunk. The ascending cervical artery arises from the inferior thyroid artery.

76. Which structure would spread infection into the anterior part of the middle nasal meatus through the frontonasal duct?

E The frontal sinus drains into the anterior part of the middle nasal meatus via the frontonasal duct or infundibulum.

43. An 8-year-old boy suffers a fracture at the base of the skull from the impact of a terrorist bomb explosion. The skull trauma includes a lesion of the vagus nerve. Damage at which of the indicated openings would injure the vagus nerve?

E Jugular foramen. The jugular foramen is a large opening that connects the posterior cranial fossa with the exterior base of the skull. It is divided into an anterior and a posterior compartment for its major contents. The anterior compartment transmits a bundle of three cranial nerves out of the cranial cavity: the glossopharyngeal nerve (CN IX), the vagus nerve (CN X), and the (spinal) accessory nerve (CN XI). The superior ganglia of the glossopharyngeal and vagus nerves are located within the jugular foramen, whereas the inferior ganglia are situated just outside it. Upon exiting the jugular foramen, the nerves immediately diverge from one another to pass to their target regions. The posterior compartment transmits the internal jugular vein. This large vessel originates at the terminal end of the sigmoid dural sinus, at the internal opening of the jugular foramen. Thus, trauma to the jugular foramen can have significant and widespread vascular and neural consequences. Choice A (Foramen ovale) is incorrect. This large, oval-shaped opening connects the middle cranial fossa with the exterior base of the skull. It transmits the mandibular division of the trigeminal nerve (CN V3) from the trigeminal ganglion into the infratemporal fossa. A small accessory meningeal artery (a branch of the maxillary artery) typically accompanies the nerve through the foramen ovale. Choice B (Foramen spinosum) is incorrect. This small foramen conveys the middle meningeal artery (a branch of the maxillary artery) and the spinous nerve (a branch of the mandibular division of the trigeminal nerve) from the infratemporal fossa into the cranium. Choice C (Foramen lacerum) is incorrect. The foramen lacerum is a large opening that connects the floor of the middle cranial fossa with the exterior base of the skull. Its irregular form gives the appearance of a roughly torn area (a laceration) in the skull, hence its name. However, in life, it is almost entirely filled with cartilage, and the true foramen is very small. The greater petrosal nerve (a parasympathetic branch of the facial nerve) traverses this foramen lacerum in passing from the floor of the middle cranial fossa to the mouth of the pterygoid canal at its anterior margin. Also, small lymphatic vessels and perhaps small emissary veins pass through this cranial foramen. Choice D (Carotid canal) is incorrect. The carotid canal is a large, obliquely running passage through the base of the skull. It carries the internal carotid artery from the deep, upper neck into the middle cranial fossa. The carotid canal merges with the internal opening of the foramen lacerum within the skull to bring the internal carotid into position at the side of the sella turcica.

"26. A 36-year-old man flips over the handlebars of his motorcycle and falls on the asphalt pavement, striking his head. He was not wearing a helmet. Although alert after the fall, he has a clear nasal discharge that tests positive for glucose. The patient most likely has a fracture of which of the following bones? (A) Ethmoid (B) Vomer (C) Sphenoid (D) Maxilla (E) Frontal"

Ethmoid A fracture of the ethmoid bone, specifically its cribiform plate, which separates the nasal cavity from the anterior cranial fossa, would enable cerebrospinal fluid (CSF), the clear discharge that tests positive for glucose, to leak from the nose. The traumatic blow to the head has broken the cribiform plate of the ethmoid bone, which caused a communication between the patient's anterior cranial fossa and nasal cavity, which is noted by the black arrow in the given sagittal CT scan. This patient presents with CSF rhinorrhea, when can lead to meningitis and other intracranial complications, and this condition can be lethal if not properly treated. The given sagittal CT shows several fracture sites, including within the cribiform plate of the ethmoid bone as well as fractures of the anterior and posterior walls of the frontal sinus. The cribiform plate is fractured in several locations, and one of these fracture sites is indicated by the white arrow in the given CT. Choice B (Vomer) is incorrect. The unpaired vomer bone forms the bony posteroinferior component of the nasal septum. So, fracturing the vomer bone would not lead to the CSF rhinorrhea presentation of this patient. Choice C (Sphenoid) is incorrect. Portions of the sphenoid bone, specifically the crest and anterior part of the sphenoid body, do form a small component of the posterior roof of the nasal cavity. When a patient presents with CSF rhinorrhea, the cribiform plate of the ethmoid bone is the most likely fracture site, which will lead to a communication between the anterior cranial fossa and the nasal cavity. The given CT was added for visual evidence of this type of fracture. It should be noted that the sphenoid bone is fractured in this individual, which is typical of this type of trauma to the anterior skull base. Remember that the thinness of the cribiform plate of the ethmoid bone makes it more susceptible to injury, so the sphenoid bone is not the best answer for this question. Choice D (Maxilla) is incorrect. The maxilla contributes to the anterolateral walls of the nasal cavity and forms most of the boundary between the nasal and oral cavities. Though it is susceptible to injury, fracturing the maxilla would not provide a communication between the anterior cranial fossa and the nasal cavity or the CSF rhinorrhea seen in this patient. Choice E (Frontal) is incorrect. The nasal spine of the frontal bone does form a small part of the roof of the nasal cavity. However, this bone is not the BEST selection for this question as it does not contribute nearly as much to the roof as the ethmoid bone. The thinness of the cribiform plate of the ethmoid bone makes it a more likely candidate to cause CSF rhinorrhea after a fracture. The given CT was added for visual evidence of a fracture to the cribiform plate of the ethmoid bone. It should be noted that the frontal bone, specifically the anterior and posterior walls of the frontal sinus, is fractured in this individual.

32. A 43-year-old man has new onset of difficulty with speaking. Examination by the ENT resident reveals problems in elevating the hyoid bone and floor of the mouth, secondary to paralysis of the posterior belly of the digastric muscle. Which of the following nerves is most likely involved? (A) Accessory nerve (B) Trigeminal nerve (C) Ansa cervicalis (D) Facial nerve (E) Glossopharyngeal nerve

Facial nerve The digastric posterior belly is innervated by the facial nerve, whereas the digastric anterior belly is innervated by the trigeminal nerve. The accessory nerve sup- plies the sternocleidomastoid and trapezius muscles. The ansa cervicalis innervates the infrahyoid (or strap) muscles. The glossopharyngeal nerve supplies the stylopharyngeus muscle.

"60. A tumor is discovered embedded in the posterior wall of the tympanic cavity in a 45-year-old man. If the tumor erodes through this wall, which of the following structures will it first encounter? (A) Internal jugular vein (B) Tympanic membrane (C) Internal carotid artery (D) Brain (E) Facial nerve"

Facial nerve The tympanic (middle ear) cavity is a small air-filled space within the petrous part of the temporal bone. Its position, shape, and relations make for challenging spatial concepts in anatomy because it is buried so deeply within the skull. It has six walls: the roof (tegmental wall), floor (jugular wall), anterior (carotid) wall, posterior (mastoid) wall, medial (labyrinthine) wall, and lateral (membranous) wall. Each wall has a close relationship to one or more significant neighboring structures. The upper part of the posterior wall contains an opening (aditus) that leads to the mastoid antrum and air cells. The distal limb of the facial canal (containing the main branch of the facial nerve) descends below the aditus, behind the posterior wall, on its way to its termination at the stylomastoid foramen. Thus, a tumor piercing the posterior wall may invade the facial canal and/or the mastoid air sinuses. Choice A (Internal jugular vein) is incorrect. The superior bulb of the internal jugular vein lies beneath the floor (jugular wall) of the tympanic cavity. Because this tumor is embedded in the posterior wall of the middle ear cavity, the internal jugular vein would not be affected. Choice B (Tympanic membrane) is incorrect. The tympanic membrane (eardrum) forms most of the lateral (membranous) wall. The epitympanic recess, situated above the tympanic membrane, completes the lateral wall. The handle of the malleus and the crossing chorda tympani nerve lie against the inside of the tympanic membrane. Because this tumor is embedded in the posterior wall of the tympanic cavity, the tympanic membrane would not be affected. Choice C (Internal carotid artery) is incorrect. The carotid canal (containing the internal carotid artery) lies outside the anterior (carotid) wall. Also, the pharyngotympanic (auditory) tube and the canal for the tensor tympani muscle open into the upper part of the anterior wall. Because this tumor is embedded in the posterior wall of the tympanic cavity, the internal carotid artery would not be affected. Choice D (Brain) is incorrect. The roof (tegmental wall) of the tympanic cavity is the tegmen tympani (roof of tympanum). This thin bony layer forms part of the floor of the middle cranial fossa, separating the dura mater and temporal lobe of the brain from the middle ear. Thus, fractures here may result in leakage of cerebrospinal fluid into the tympanic cavity and subsequently into the nasopharynx (via the pharyngotympanic tube) or external acoustic meatus (if the tympanic membrane is ruptured). The sixth wall (medial; labyrinthine) separates the middle ear from the internal ear. Its main features are the promontory, oval window, and round window.

Mandibulofacial dysostosis (Treacher-Collins syndrome) is a developmental disorder characterized by craniofacial defor- mities, including malformed or absent ears, zygomatic and mandibular hypoplasia, and downward slanting eyes exhibit- ing ptosis of the lateral eyelids, as shown in the photo. This condition is the result of lack of migration of neural crest cells into what pharyngeal arch? (A) First pharyngeal arch (B) Second pharyngeal arch (C) Third pharyngeal arch (D) Fourth pharyngeal arch (E) Fifth pharyngeal arch

First pharyngeal arch Mandibulofacial dysostosis (Treacher-Collins syndrome) affects derivatives of the first pharyngeal arch, specifically the migration of the neural crest cells into the arch. The cranial neural crests of the first pharyngeal arch are responsible for formation of the mandible, malleus, incus, squamous portion of the temporal bone, temporal bone, palatine bone, and vomer. As the figure shows, this patient exhibits micrognathia (small lower jaw), malar (zygomatic) hypoplasia, and malformed external ears, and this craniofacial deformities clearly indicate problems with the neural crest migration of the first pharyngeal arch. Mandibulofacial dysostosis is seen in 1 in 10,000 births. Choice B (Second pharyngeal arch) is incorrect. Neural crest cells of the second pharyngeal arch are responsible for formation of the stapes, styloid processes, and the upper body and lesser horns of the hyoid bone. Malformation of these structures derived from the second pharyngeal arch would not lead to the characteristic craniofacial deformities seen in a patient with mandibulofacial dysostosis (Treacher-Collins syndrome). Choice C (Third pharyngeal arch) is incorrect. Neural crest cells of the third pharyngeal arch are responsible for formation of the lower body and greater horns of the hyoid bone. Malformation of these third pharyngeal arch structures would not lead to the characteristic craniofacial deformities seen in a patient with mandibulofacial dysostosis. Choice D (Fourth pharyngeal arch) is incorrect. Neural crest cells of the fourth pharyngeal arch are not responsible for any skeletal elements. Therefore, failure of the neural crest cell migration of the fourth pharyngeal arch structures would not lead to any craniofacial deformities, and this option can be easily eliminated. Choice E (Fifth pharyngeal arch) is incorrect. The fifth pharyngeal arch only exists transiently in human embryologic growth and development. No structures are derived from the fifth pharyngeal arch in adults, so this option can be easily eliminated.

66. A 2-year-boy presents with midfacial and mandibular hypoplasia, cleft palate, deformed external ear, and defect in hearing. Which of the following embryonic structures is most likely developed abnormally? (A) First pharyngeal arch (B) Second pharyngeal arch (C) Third pharyngeal arch (D) Fourth pharyngeal arch (E) Sixth pharyngeal arch

First pharyngeal arch The patient's abnormal appearance results from abnormal development of the first pharyngeal arch because the first pharyngeal arch develops into muscles of mastication, mylohyoid, digastric anterior belly, tensor veli palatini, tensor tympani, max- illa, mandible, malleus, incus, zygomatic bone, temporal bone, palatine bone, vomer, and sphenomandibular ligament.

72. A 3-year-old girl is admitted to the hospital with pain and hearing defect. An MRI examination reveals that she has developmental defects in the auditory tube and middle ear cavity. Which of the following pharyngeal pouches is most likely developed abnormally? (A) First pouch (B) Second pouch (C) Third pouch (D) Fourth pouch (E) Second and fourth pouches

First pouch The first pharyngeal pouch gives rise to the auditory tube and middle ear cavity. The second pouch forms the palatine tonsils. The third pouch gives rise to the infe- rior parathyroid gland and thymus. The fourth pouch develops into the superior parathy- roid gland and ultimobranchial body of the thyroid.

75. A physician examines a 53-year-old woman and notes deviation of the uvula to the right and asymmetry in the elevation of the soft palate, with the palatal arch of the left side sagging when compared to the right. The muscles involved in these abnormal findings are most likely derived from the mesoderm of what pharyngeal arch? (A) First (B) Second (C) Third (D) Fourth (E) Sixth

Fourth This patient is exhibiting atrophy of the muscles on the left side of the soft palate, which are derived from the mesoderm of the fourth pharyngeal arch. In this patient, the palatal arch droops on the affected (left) side, and the uvula deviates to the unaffected (right) side as a result of the unopposed action of the intact (contralateral) muscles acting on the soft palate. This patient presentation would be indicative of a left vagus nerve (CN X) injury. Remember that the pharyngeal branches of CN X innervate all of the musculature of the soft palate, except the tensor veli palatini (tensor of the soft palate), which is innervated by the mandibular division of the trigeminal nerve (CN V3). Besides forming most of the muscles of the soft palate, the mesoderm of the fourth pharyngeal arch would give rise to the muscles of the pharynx (with the exception of the stylopharyngeus muscle) and the cricothyroid, a muscle of the larynx. Choice A (First) is incorrect. The muscles derived from the mesoderm of the first pharyngeal arch include four muscles of mastication (temporalis, masseter, lateral pterygoid, and medial pterygoid) and four additional muscles: Mylohoid, Anterior belly of the Digastric, Tensor Tympani, and Tensor Veli Palatini (mnemonic = "MATT"). The tensor veli palatini muscle is the only muscle of the soft palate derived from the first pharyngeal arch. However, damage to the first pharyangeal arch musculature would not affect elevation of the soft palate or deviation of the uvula. Choice B (Second) is incorrect. The muscles derived from the mesoderm of the second pharyngeal arch include the muscles of facial expression and three additional muscles: posterior belly of digastric, stylohyoid, and stapedius. None of these listed muscles are associated with the soft palate, so they play no role in this patient's presentation. Choice C (Third) is incorrect. The only muscle derived from the mesoderm of the third pharyngeal arch is the stylopharyngeus, a muscle of the pharynx. The stylopharyngeus is the only muscle innervated by the glossopharyngeal nerve (CN IX). Because this muscle is not associated with the soft palate, it plays no role in this patient's presentation. Choice E (Sixth) is incorrect. The muscles derived from the mesoderm of the sixth pharyngeal arch include the intrinsic muscles of the larynx and the skeletal muscle of the upper esophagus. Because these muscles are not involved with the soft palate, they play no role in this patient's presentation.

61. A 71-year-old man suffers from a known benign tumor in the pterygoid canal. Which of the following nerve fibers could be injured by this condition? (A) Postganglionic parasympathetic fibers (B) Taste fibers from the epiglottis (C) General somatic afferent (GSA) fibers (D) Preganglionic sympathetic fibers (E) General visceral afferent fibers

General visceral afferent fibers The nerve of the pterygoid canal (vidian nerve) contains taste (special visceral afferent [SVA]) fibers from the palate, GVA fibers, postganglionic sympathetic fib- ers, and preganglionic parasympathetic fibers.

"62. During a fight between two construction workers, one man strikes the other with a hammer at the pterion of the skull. Which of the following bones may be fractured? (A) Zygomatic process of the temporal bone (B) Greater wing of the sphenoid bone (C) Mastoid process of the temporal bone (D) Lateral pterygoid plate of the sphenoid bone (E) Coronoid process of the mandible"

Greater wing of the sphenoid bone The pterion is a significant craniometric landmark point in the temporal fossa on the lateral aspect of the skull. It is the roughly H-shaped junction of four bones: greater wing of the sphenoid, frontal, parietal, and squamous parts of the temporal. The pterion is clinically significant in that the bones here are relatively thin and susceptible to fracture from impact. Furthermore, the anterior branch of the middle meningeal artery typically lies tightly grooved against the interior of the skull at this point. The middle meningeal artery is the major vessel supplying the dura mater and the bones of the cranial vault. It can be readily ruptured in trauma to the pterion, and the middle meningeal artery is the primary vessel implicated in epidural hemorrhage. Choice A (Zygomatic process of the temporal bone) is incorrect. The zygomatic process of the temporal bone is an anterior projection articulating with the zygomatic bone. Together, these two components form the zygomatic arch. Choice C (Mastoid process of the temporal bone) is incorrect. The mastoid process of the temporal bone is a breast-like projection extending inferior from the temporal bone behind the external acoustic meatus. It forms part of the posterior boundary of the infratemporal fossa. Additionally, it is the attachment site for certain neck muscles (e.g., sternocleidomastoid) and is largely hollowed by the mastoid air cells. Choice D (Lateral pterygoid plate of the sphenoid bone) is incorrect. The lateral pterygoid plate is a thin, winglike, inferior extension of the sphenoid bone. It forms the medial (deep) boundary of the infratemporal fossa and provides attachment for portions of the medial and lateral pterygoid muscles. Choice E (Coronoid process of the mandible) is incorrect. The coronoid process of the mandible is a thin, blade-like, superior projection from the anterior aspect of the ramus of the mandible. It serves as the insertion area for the large temporalis muscle, a muscle of mastication.

A 22-year-old man receives a stab wound in the left anterior cervical region, at the C2 vertebral level. The wound was 3 cm deep and located anterior to the sternocleidomastoid muscle (SCM) and superior to the greater horn of the hyoid bone. During a postoperative examination, the patient displays dysarthria, or difficulty speaking. Which of the following structures is most likely damaged? (A) Hypoglossalnerve (B) Accessorynerve (C) Mandibular division of trigeminal nerve (D) Lingual branch of glossopharyngeal nerve (E) Roots of the brachial plexus"

Hypoglosaal nerve The left hypoglossal nerve (CN XII) is located in the anterior cervical region at the location of the stab wound. CN XII innervates all of the intrinsic muscles of the tongue and most of its extrinsic muscles with the lone exception being the palatoglossus muscle, innervated by the vagus nerve (CN X). Therefore, damage to the right CN XII would produce dysarthria, or difficult speaking, which was seen in this patient due to the loss of innervation to the tongue musculature. Though not noted in this patient, damage to the hypoglossal nerve also causes ipsilateral deviation of the tongue due to the unopposed muscular contractions of the contralateral genioglossus muscle. The mnemonic "The tongue licks the wound" will help you remember that the tongue deviates to the ipsilateral side in a lower motor neuron lesion of CN XII. Choice B (Accessory nerve) is incorrect. The accessory nerve (CN XI) passes deep to the SCM and to the investing layer of deep cervical fascia and courses posterolaterally into the lateral cervical region (posterior triangle of neck). Cutting this nerve would not produce the dysarthria displayed in this patient because CN XI innervates only the SCM and the trapezius muscle, which are primarily involved in head and shoulder movements. Also, the accessory nerve is not located in the anterior cervical region in the location of the stab wound. Choice C (Mandibular division of trigeminal nerve) is incorrect. The mandibular (third) division of the trigeminal nerve (CN V3) is the only division of the trigeminal nerve (CN V) that supplies motor innervation. It supplies the muscles derived from the mesoderm of the first pharyngeal arch, including the four muscles of mastication (temporalis, masseter, lateral pterygoid, and medial pterygoid) and four additional muscles: Mylohoid, Anterior belly of the Digastric, Tensor Tympani, and Tensor Veli Palatini (mnemonic = "MATT"). These muscles play no role in the dysarthria noted in this patient, and CN V3 is not located in close proximity to the stab wound. Choice D (Lingual branch of glossopharyngeal nerve) is incorrect. The lingual branch of the glossopharyngeal nerve (CN IX) enters the posterior one third of the tongue to provide general sensation and taste to the region. It does not provide any motor innervation, so this nerve would not be involved with the dysarthria noted in this patient. Moreover, due to the location of the knife wound, this nerve would not be affected. Choice E (Roots of the brachial plexus) is incorrect. The roots of the brachial plexus represent the anterior (ventral) rami of C5-8 and T1, and these nerves emerge between the anterior and middle scalene muscles in the lateral cervical region (posterior triangle of neck). The roots of the brachial plexus are located too inferior to be damaged by the knife.

"44. During a difficult childbirth, a physician uses obstetric forceps to grip the infant's head as an aid to extracting her from the birth canal. However, the forceps are misapplied at the right stylomastoid foramen and crush its contents at the opening of the foramen. Which of the following ipsilateral deficits is the baby most likely to suffer? (A) Reduced blood flow to the inner ear (B) No sensation in the external acoustic meatus (C) Inability to close the eyelids (D) Lack of taste on the body of the tongue (E) Inability to tense the eardrum"

Inability to close the eyelides The stylomastoid foramen is located in a well-protected position between the styloid and mastoid processes, at the base of the skull. It is the terminal opening of the facial canal, transmitting the main branch of the facial nerve out of the petrous temporal bone to the exterior base of the skull. From there, the facial nerve sends sensory branches to the external ear, motor branches to small muscles in the upper neck (stylohyoid and posterior belly of the digastric), and terminal motor branches to the facial muscles (muscles of facial expression). However, the mastoid process is not developed at birth, leaving the stylomastoid foramen and the emerging facial nerve exposed. Undue pressure applied to the mastoid area (as from misapplied obstetric forceps) may entrap and damage the facial nerve. In this case, crush injury to the facial nerve would produce unilateral paralysis of the facial muscles, including the orbicularis oculi. This sphincter-like muscle surrounds the orbit, acting to close the eyelids. Choice A (Reduced blood flow to the inner ear) is incorrect. Vascular supply to the inner ear is provided by small labyrinthine vessels that accompany the facial and vestibulocochlear nerves through the internal acoustic meatus. Supply to the external ear is mainly from branches of the posterior auricular and superficial temporal arteries. The posterior auricular arterial branching is susceptible to damage in this scenario. Supply to the middle ear is mainly from branches of the maxillary artery. These vessels are unlikely to be affected by compression around the incipient mastoid process. Choice B (No sensation in the external acoustic meatus) is incorrect. The external ear receives afferent innervation from a plethora of cranial nerves, including trigeminal (CN V), facial (CN VII), vagus (CN X), and possibly even the glossopharyngeal (CN IX). The trigeminal and vagus are the major nerves, supplying most of the sensory fibers to this region, including the external acoustic meatus. The facial nerve provides relatively small contributions to the sensation of the external acoustic meatus because its sensory innervation is concentrated on the auricle. Choice D (Lack of taste on the body of the tongue) is incorrect. The facial nerve provides taste fibers to the body (anterior two thirds) of the tongue and parasympathetic fibers to the oral floor via its chorda tympani branch. However, the chorda tympani nerve leaves the facial nerve within the facial canal, traverses the tympanic (middle ear) cavity, and exits the base of the skull near the temporomandibular joint (TMJ). Therefore, this forceps injury would not affect the taste or parasympathetic functions governed by the facial nerve. Choice E (Inability to tense the eardrum) is incorrect. The tensor tympani muscle attaches to the handle of the malleus, acting to pull this ear ossicle medially and tense the tympanic membrane (eardrum). This muscle is innervated by the mandibular division of the trigeminal nerve (CN V3). The stapedius muscle attaches to the stapes, acting to tighten the oval window. This muscle is controlled by the facial nerve, via a branch off the facial canal, proximal to the stylomastoid foramen.

"34. A 78-year-old man presents with signs of reduced blood flow into the right side of his face. His physician wishes to take a pulse of the facial artery on both sides to help evaluate the situation. The pulse of the facial artery can be readily palpated at which of the following locations? (A) Lateral side of the body of the hyoid bone (B) Inferior edge of the zygomatic arch (C) Apex of the zygomatic bone (D) Lateral surface of the nasal bone (E) Inferior margin of the body of the mandible"

Inferior margin of the body of the mandible After originating from external carotid artery, the facial artery ascends in the upper neck deep to the posterior belly of the digastric and stylohyoid muscles and the submandibular gland. The facial artery then crosses the body of the mandible anterior to the masseter muscle to enter the face. Its pulse can be readily palpated at the point where it crosses the inferior margin of the body of the mandible, at the anterior border of the masseter muscle. The artery continues ascending across the face, taking a tortuous (or winding) path relatively close to the angle of the mouth. The folding of the vessel allows it to accommodate being stretched during wide opening of the mouth. It gives branches to the lips and side of the nose, finally terminating at the medial canthus (angle) of the eye. Choice A (Lateral side of the body of the hyoid bone) is incorrect. The facial artery arises from the external carotid artery in the upper neck. It courses superior to the hyoid bone. Choice B (Inferior edge of the zygomatic arch) is incorrect. The transverse facial artery arises within the parotid gland as a small branch of the superficial temporal artery. As it runs anterior, the transverse facial artery crosses the masseter muscle and is located just above the parotid duct and below the zygomatic arch. Choice C (Apex of the zygomatic bone) is incorrect. This bony feature is the point of the "cheek bone." No significant vessels reside at this location, so taking a pulse at the apex of the zygomatic bone is not possible. Choice D (Lateral surface of the nasal bone) is incorrect. As it ascends across the face, that facial artery gives branches to the ala and side of the nose (lateral nasal artery) and its terminal branch to the medial angle (canthus) of the eye (angular artery). However, these vessels are small and do not normally give pulse points.

7. A 26-year-old singer visits her physician—an ear, nose, and throat (ENT ) surgeon—and complains of changes in her voice. A laryngoscopic examination demonstrates a lesion of the superior laryngeal nerve, causing weakness of which of the following muscles? (A) Inferior pharyngeal constrictor (B) Middle pharyngeal constrictor (C) Superior pharyngeal constrictor (D) Thyroarytenoid (E) Thyrohyoid

Inferior pharyngeal constrictor The external laryngeal branch of the superior laryngeal nerve supplies the cricothyroid and inferior pharyngeal constrictor muscles. The superior, middle, and inferior pharyngeal constrictors are innervated by the vagus nerve through the pharyngeal plexus. The recurrent (or inferior) laryngeal nerve supplies the thyroarytenoid muscle and the C1 via the hypoglossal nerve supplies the thyrohyoid muscle.

40. A high school basketball player experiences a sudden difficulty in breathing and is brought to an emergency department. When a low tracheotomy is performed below the isthmus of the thyroid, which of the following vessels may be encountered? (A) Inferior thyroid artery (B) Inferior thyroid vein (C) Costocervical trunk (D) Superior thyroid artery (E) Right brachiocephalic vein

Inferior thyroid vein A low tracheotomy is a surgical incision of the trachea through the neck and below the isthmus of the thyroid gland. The inferior thyroid veins drain the thyroid gland, descend in front of the trachea, and enter the brachiocephalic veins. Consequently, these veins are closely associated with the isthmus of the thyroid gland. Other blood vessels are not closely related with the front of the trachea and the isthmus of the thyroid gland.

64. A 12-year-old boy has difficulty in breathing because he is choking on food. A school nurse performs the Valsalva maneuver to expel air from his lungs and thus dislodge the food. When that fails, she performs a needle cricothyrotomy, which would open into which of the following regions? (A) Rima glottidis (B) Laryngeal vestibule (C) Laryngeal ventricle (D) Infraglottic cavity (E) Piriform recess

Infraglottic cavity The infraglottic cavity extends from the rima glottidis to the lower border of the cricoid cartilage. The rima glottidis is the space between the vocal folds and aryten- oid cartilages. The vestibule extends from the laryngeal inlet to the vestibular folds. The ventricle extends between the vestibular fold and the vocal fold. The piriform recess is a pear-shaped fossa in the wall of the laryngopharynx lateral to the arytenoid cartilage.

47. A 76-year-old man with swallowing difficulties undergoes imaging for a possible mass. The CT scan image at the level of the cricothyroid ligament in his neck should show which of the following structures? (A) Inferior laryngeal nerves (B) External carotid arteries (C) Inferior thyroid veins (D) Thyrocervical trunks (E) Internal laryngeal nerves

Interior laryngeal nerves A computed tomography (CT) scan through the cricothyroid ligament shows the inferior laryngeal nerves, which are the terminal portion of the recurrent laryn- geal nerves above the lower border of the cricoid cartilage. The external carotid arteries and the internal laryngeal nerves lie above the cricothyroid ligament, and the inferior thy- roid veins and the thyrocervical trunks lie below the ligament.

53. A 21-year-old woman presents to her physician with a swelling on her neck. On examination, she is diagnosed with an infection within the carotid sheath. Which of the following structures would be damaged? (A) Vagus nerve and middle cervical ganglion (B) Internal carotid artery and recurrent laryngeal nerve (C) Internal jugular vein and vagus nerve (D) Sympathetic trunk and common carotid artery (E) External carotid artery and ansa cervicalis

Internal jugular vein and vagus nerve The carotid sheath contains the internal jugular vein, vagus nerve, and common and internal carotid arteries. The recurrent laryngeal nerve lies in a groove between the trachea and esophagus. The sympathetic trunk, with superior and middle cervical ganglia, lies behind the carotid sheath. The external carotid artery is not con- tained within the carotid sheath. The ansa cervicalis lies superficial to or within the carotid sheath.

2. A 27-year-old woman with a goiter comes to the hospital for surgical treatment. The surgeon must ligate the superior laryngeal artery before surgically resecting the goiter, so care must be taken to avoid injury to which of the following nerves? (A) External laryngeal nerve (B) Internal laryngeal nerve (C) Superior laryngeal nerve (D) Hypoglossal nerve (E) Vagus nerve

Internal laryngeal nerve The internal laryngeal nerve accompanies the superior laryngeal artery, whereas the external laryngeal nerve accompanies the superior thyroid artery. The supe- rior laryngeal, hypoglossal, and vagus nerves are not closely associated with the superior laryngeal artery.

30. During a carotid endarterectomy of a 57-year-old man who suffered a stroke, the carotid sinus is damaged. A third-year medical student in surgical rotation notices that the injured structure: (A) Is located at the origin of the external carotid artery (B) Is innervated by the facial nerve (C) Functions as a chemoreceptor (D) Is stimulated by changes in blood pressure (E) Communicates freely with the cavernous sinus

Is stimulated by changes in blood pressure The carotid sinus, a spindle-shaped dilatation of the origin of the inter- nal carotid artery, is a pressoreceptor that is stimulated by changes in blood pressure. The carotid sinus is at the origin of the internal carotid artery and is innervated by the carotid sinus branch of the glossopharyngeal nerve and nerve to the carotid body of the vagus nerve. It is not a venous sinus and thus does not communicate with the cavernous sinus. The carotid body functions as a chemoreceptor.

52. A 58-year-old woman comes to a hospital and complains of progressive loss of voice, numbness, loss of taste on the back part of her tongue, and difficulty in shrugging her shoulders. Her MRI scan reveals a dural meningioma that compresses the nerves leaving the skull. These nerves leave the skull through which of the following openings? (A) Foramen spinosum (B) Foramen rotundum (C) Internal auditory meatus (D) Jugular foramen (E) Foramen lacerum

Jugular foramen A loss of voice is due to an injury to the recurrent laryngeal nerve of the vagus nerve; numbness and loss of taste on the posterior part of the tongue is due to a lesion of the glossopharyngeal nerve; an inability to shrug the shoulder is due to damage of the accessory nerve. These three cranial nerves exit the skull through the jugular foramen. The foramen spinosum transmits the middle meningeal artery. The foramen rotundum transmits the maxillary division of the trigeminal nerve. The internal auditory meatus transmits the facial and vestibulocochlear nerves. The foramen lacerum transmits nothing, but its upper part is traversed by the internal carotid artery with sympathetic nerve plexus.

A 27-year-old man comes to his doctor complaining that the floor of his mouth is painful and swollen. Bimanual palpation and later computed tomography show an 8-mm stone (sialolith) in the right submandibular salivary duct, as indicated by the arrow in the figure. If the submandibular duct is partially obstructed, where would the physician observe salivation in the oral cavity after applying pressure to the right submandibular gland? (A) Second maxillary molar tooth (B) Oral vestibule (C) Palatine mucosa (D) Sublingual fold (E) Lingual frenulum

Lingual frenulum The openings of the submandibular ducts reside on either side of the lingual frenulum, and the saliva produced by the submandibular glands drains through the sublingual caruncles (or papillae), openings at the base of the lingual frenulum. By applying pressure to the right submandibular gland, the physician would look at the lingual frenulum for the salivary secretions of the submandibular glands. If no salivation is observed, then the right submandibular salivary duct is completely obstructed. Choice A (Second maxillary molar tooth) is incorrect. The duct of the parotid gland drains saliva into the oral vestibule via a small opening opposite the second maxillary molar teeth on either side of the oral cavity. In this case, the physician needs to look for the drainage of the submandibular duct, which is at the sublingual caruncle located within the base of the lingual frenulum. Choice B (Oral vestibule) is incorrect. The duct of the parotid gland drains saliva into the oral vestibule via a small opening opposite the second maxillary molar teeth on either side of the oral cavity. In this case, the physician needs to look for the drainage of the submandibular duct, which is at the sublingual caruncle located within the base of the lingual frenulum. Choice C (Palatine mucosa) is incorrect. The ducts of the palatine glands reside deep to the mucosa of the hard palate, and they drain their mucous secretions into the superior aspect of the oral cavity. In this case, the physician needs to look for the drainage of the submandibular duct, which is at the sublingual caruncle located within the base of the lingual frenulum. Choice D (Sublingual folds) is incorrect. The sublingual folds lie in the floor of the mouth and represent the location of the drainage of the sublingual glands. The small sublingual ducts conduct saliva into the oral cavity through numerous openings within the sublingual folds.

"31. A 17-year-old girl uses Accutane, an acne drug implicated in interfering with normal development of the facial primordia, early in her unexpected pregnancy. Examination of the newborn reveals the craniofacial defect seen in the photo. In this case, the defect is related to failure of fusion of what craniofacial processes? (A) Maxillary and medial nasal processes (B) Opposite maxillary processes (C) Medial and lateral nasal processes (D) Maxillary and lateral nasal processes (E) Opposite medial nasal processes"

Maxillary and medial nasal processes The face and palate are formed from the differentiation, growth, and merging of five facial primordia (facial prominences; facial swellings; facial processes): the single frontal (frontonasal) prominence and the paired maxillary and mandibular prominences (see figure on next page). The frontal prominence secondarily gives rise to paired nasal placodes and their bordering medial and lateral nasal prominences. Failure of the facial prominences to merge and fuse results in gaps (clefts) left remaining between primordial tissue zones. These gaps are the basis for palatofacial clefts of varying location and severity. The infant in this case suffers an anterior palatal cleft (cleft lip; harelip). Anterior clefts are located anterior to the incisive foramen. That location plus its position lateral to the midline indicates the defect was caused by incomplete fusion of the maxillary and medial nasal processes. Here, the defect is relatively small and unilateral. In more severe cases, the cleft may extend deeper and/or occur bilaterally. Cleft lip is a relatively common condition (1:1,000 births) that occurs more often in males and has an incidence that increases slightly with maternal age. Unilateral cleft lip is the most common craniofacial congenital defect. Choice B (Opposite maxillary processes) is incorrect. The opposite maxillary processes normally develop horizontal palatine shelves that fuse with each another in the midline to form the secondary palate (i.e., the bulk of the hard and soft palate). Defects in this fusion are termed posterior clefts (cleft palate) because they are located posterior to the incisive foramen. They vary in degree from relatively minor (cleft of only the uvula) to severe (cleft of the entire secondary palate). Posterior clefts occur less often than anterior clefts (1:2,500 births), occur more often in females, and are not related to maternal age. Choice C (Medial and lateral nasal processes) is incorrect. The medial and lateral nasal processes border each nasal placode. The tissue between each pair of nasal processes invaginates and canalizes to eventually form a nasal passage (i.e., the airway from naris to choana). Thus, the nasal processes fuse with each other only at their upper and lower margins to allow formation of the nasal passages. Choice D (Maxillary and lateral nasal processes) is incorrect. The maxillary prominence normally overgrows the nasolacrimal groove to fuse with its matching lateral nasal prominence. This development causes the nasolacrimal groove to sink into the face, where it canalizes and forms the nasolacrimal duct. Failure of this fusion is rare. When occurring, the result is an oblique facial cleft. In this craniofacial malformation, the defect runs from the medial canthus of the eye into the upper lip, with the nasolacrimal duct typically exposed to the surface. Choice E (Opposite medial nasal processes) is incorrect. The two medial nasal processes normally fuse in the midline to form the bridge of the nose, the nasal septum, and the intermaxillary segment of the face (philtrum of the upper lip; premaxilla; primary palate). Failure in this fusion is rare. The resultant malformation may be a midline grooved or cleft (bifid) nose and/or defects in the intermaxillary segment of the face. Such conditions may indicate a more extensive loss of midline tissue (including neural tissue), characteristic of holoprosencephaly.

"30. A 31-year-old woman with an ongoing, long-term history of alcoholism becomes pregnant. The embryo she is carrying suffers a neural crest insufficiency during the critical period of development. Which of the following structures is most likely to be malformed as a result of this condition? (A) Laryngeal cartilages (B) Parietal bones (C) Maxillary bones (D) Common carotid arteries (E) Thyroid gland"

Maxillary bones Neural crest cells arise in association with the neuroectoderm of the developing brain, and migrate into the pharyngeal arches, around the forebrain, and into the facial region. In these regions, neural crest cells account for numerous skeletal and other structures, including cartilage, tendon, dentin, dermis, sensory ganglia, and the arachnoid and pia mater. Therefore, development and migration of sufficient numbers of neural crest cells are critical for formation of much of the head and neck, especially the craniofacial region. Because neural crest cells also contribute significantly to heart formation, many infants with craniofacial defects also exhibit cardiac malformations. Unfortunately, neural crest cells are highly sensitive to environmental teratogens such as alcohol and retinoic acid. Overexposure to these substances (e.g., in fetal alcohol syndrome) may reduce production, kill, and/or limit the migration of the neural crest cells into their target regions. Neural crest cells form virtually all the facial skeleton, including the frontal, maxillary, zygomatic, squamous temporal, and mandibular bones, plus other smaller bones. Neural crest insufficiency is the major cause of palatofacial clefts that result from failure of fusion of facial primordia. Choice A (Laryngeal cartilages) is incorrect. Although neural crest cells form most of the skeletal structures (bone and cartilage) derived from the pharyngeal arches, the laryngeal cartilages are notable exceptions, derived from the lateral plate mesoderm of the fourth and sixth arches. Choice B (Parietal bones) is incorrect. Most of the cranial vault (including the parietal, occipital, and petrous portion of the temporal bones) and the cranial floor are formed from paraxial mesoderm (somites and somitomeres). Choice D (Common carotid arteries) is incorrect. In general, the aortic arches that run within the pharyngeal arches are derived from the mesoderm of the pharyngeal arches, along with the muscles that originate there. Thus, the arteries derived from the aortic arches, including the common carotid arteries, have mesodermal origins. Choice E (Thyroid gland) is incorrect. The thyroid gland originates as an epithelial (endodermal) growth in the floor of the embryonic pharynx. It proliferates as a single, ventromedian diverticulum (thyroid diverticulum) off the pharynx, and descends into the neck.

28. A 13-year-old boy competing in a motocross competition falls from his bike and sustains massive head injuries. Which of the following cavities are separated from the middle cranial fossa by a thin layer of bone? (A) Auditory tube and bony orbit (B) Middle ear cavity and sphenoid sinus (C) Sigmoid sinus and frontal sinus (D) Sphenoid sinus and ethmoid sinus (E) Maxillary sinus and middle ear cavity

Middle ear cavity and sphenoid sinus The middle ear cavity is separated from the middle cranial fossa by the tegmen tympani, a thin plate of the petrous part of the temporal bone. A part of the roof of the sphenoid bone forms the floor of the hypophyseal fossa. The other pairs of sinuses or bony cavities are not separated from the middle cranial cavity.

"50. In the process of removing cervical lymph nodes during a radical neck dissection, a surgeon mistakenly lesions the ansa cervicalis. Which of the following deficits may occur? (A) Decreased blood flow to the larynx (B) Lymphedema in the carotid triangle of the neck (C) Reduced sensation in the skin over the posterior triangle of the neck (D) Paralysis of several infrahyoid (strap) muscles (E) Paralysis of the intrinsic laryngeal muscles"

Paralysis of several infra hyoid (strap) muscles The ansa cervicalis is a motor nerve loop derived from the cervical plexus. It is formed by the union of its two parts: the superior root (descendens hypoglossi, from C1) and the inferior root (descendens cervicalis, from C2 and C3). This delicate structure normally lies on the superficial surface of the carotid sheath, with its inferior margin at about the level of the cricoid cartilage. The ansa cervicalis innervates most of the infrahyoid (strap) muscles (omohyoid, sternohyoid, sternothyroid). The exception is the thyrohyoid muscle being supplied by C1 via the hypoglossal nerve. Choice A (Decreased blood flow to the larynx) is incorrect. The ansa cervicalis is a motor nerve route to skeletal muscles in the neck. It does not carry autonomic fibers that influence vasomotor control. These fibers are derived from the cervical sympathetic chain. Choice B (Lymphedema in the carotid triangle of the neck) is incorrect. The ansa cervicalis is located mainly in the carotid triangle. However, it does not regulate lymphatic flow. Removal of lymph nodes in this region may produce some degree of lymphedema due to removal of the lymphatic channels, but that result is not related to lesion of the ansa cervicalis. Choice C (Reduced sensation in the skin over the posterior triangle of the neck) is incorrect. The ansa cervicalis does not convey cutaneous sensation. The cutaneous branches of the cervical plexus emerge together from under the posterior border of the sternocleidomastoid muscle, at a point termed the "punctum nervosum" (nerve point of the neck). From this location, the cutaneous nerves of the cervical plexus distribute across the skin of the anterior and posterior triangles of the neck, posterior scalp, lower face, and anterior shoulder. Choice E (Paralysis of the intrinsic laryngeal muscles) is incorrect. The intrinsic muscles controlling the larynx are all supplied by branches of the vagus nerve (CN X). Most are innervated by the recurrent laryngeal nerve of CN X. The cricothyroid muscle is supplied by the external laryngeal nerve off the superior laryngeal nerve of CN X.

"51. Identify the structure indicated with the letter "X" in this X-ray of the lateral neck. (A) Vertebral body (B) Pedicle (C) Intervertebral disc space (D) Superior articular process (E) Inferior articular process"

Pedicle The image is a lateral plane film (X-ray) of the cervical component of the vertebral column. This type of radiological imaging is used commonly for general evaluation of the neck, especially in severe neck injuries where fractures may be suspected. The pedicle is the constricted "foot" of the vertebral arch that links the neural arch with the body of a vertebra, and it is identified by the white leader line from the letter "B" in the labeled X-ray provided. In this view, the opposite pedicles are superimposed upon one another. Thus, these small structures produce a distinct radiodensity (radiopacity) in the film. Choice A (Vertebral body) is incorrect. The body is the large, block-like structure at the anterior aspect of the column, and it is identified by the white leader line from the letter "A" in the labeled X-ray provided. The margins of the cervical bodies typically present a smooth curvature on both the anterior and posterior aspects. Deviations of the normal curvature of the vertebral bodies may suggest a possible fracture and/or torn ligaments. Choice C (Intervertebral disc space) is incorrect. This radiolucent area, identified by the white leader line from the letter "C" in the labeled X-ray provided, is located between the vertebral bodies and is occupied by the intervertebral disc. The height of this space decreases with degeneration of the disc (from either pathology or aging), thus approximating the vertebral bodies. Choice D (Superior articular process) is incorrect. Because the articular processes in the cervical region are positioned at oblique inclinations, they appear as seemingly sharply pointed structures in the lateral view, which can be misinterpreted as displaced bone fragments. The superior articular process, identified by the white leader line from the letter "D" in the labeled X-ray provided, is an upwardly projecting element, and it articulates with the inferior articular process of the more superior vertebra to create a zygapophysial (or facet) joint between the vertebral arches. Choice E (Inferior articular process) is incorrect. The inferior articular process, identified by the white leader line from the letter "E" in the labeled X-ray, can be seen projecting inferior in the lateral view. The inferior articular process articulates with the superior articular process of the more inferior vertebra to create a zygapophysial (or facet) joint between the vertebral arches. The articular processes are outgrowths of the vertebral arch at the junction of the pedicle and the lamina.

"63. The given photo shows a superior view of the head of a baby boy with scaphocephaly. Which of the following sutures closed prematurely in this infant? (A) Coronal (B) Sagittal (C) Lambdoid (D) Metopic (E) Squamous"

Sagittal Premature closure of the sagittal suture leads to scaphocephaly (G: boat-shaped head), or dolichocephaly, which presents with a disproportionately long and narrow skull. This marked increase in head length with narrowed width is particularly common in infants who are born prematurely. Surgical intervention is needed to treat scaphocephaly to remove bone from the sagittal suture. Craniosynostosis is the term that refers generally to the premature fusion of the cranial sutures. Choice A (Coronal) is incorrect. Premature closure of the coronal suture leads to brachycephaly (G: short head), which involves a disproportionately wide skull with a short occipitofrontal diameter. Babies with brachycephaly have a characteristic square-shaped skull due to the short occipitofrontal diameter seen following premature closure of the coronal suture. Brachycephaly is more common in females, and surgical intervention can be implemented to remove bone from the coronal suture. If only one side of the coronal suture closes prematurely, the infant presents with an asymmetric cranium, a condition known as plagiocephaly. In this baby boy, the long and narrow shape of the skull indicates premature closure of the sagittal suture. Choice C (Lambdoid) is incorrect. The lambdoid suture is a dense, fibrous connective tissue joint located on the back of the skull that connects the occipital bone with the posterior aspect of the parietal bone and petrous portion of the temporal bone. If only one side of the lambdoid suture closes prematurely, the infant presents with a twisted and asymmetric cranium, a condition known as plagiocephaly. In this baby boy, the long and narrow shape of the skull indicates premature closure of the sagittal suture. Choice D (Metopic) is incorrect. The metopic (or frontal) suture is a dense, fibrous connective tissue joint located between the two halves of the frontal bone. This suture usually begins fusion of the frontal bones at age 2 and disappears by age 6. If the metopic suture is not present at birth, trigonacephaly, a type of craniosynostosis, results in a keel-shaped deformity. In this baby boy, the long and narrow shape of the skull indicates premature closure of the sagittal suture. Choice E (Squamous) is incorrect. The squamous suture is a dense, fibrous connective tissue joint, which arches between the parietal bone and the squamous portion of the temporal bone, reaching anterior to the pterion. This suture is usually the last cranial suture to close around ages 35 to 39. In this baby boy, the long and narrow shape of the skull indicates premature closure of the sagittal suture.

"33. A 25-year-old woman notes the pictured asymmetry in her neck when she tenses the skin of her inferior face and neck. This asymmetry may be due to a limited mesodermal migration in which of the following embryonic structures? (A) First pharyngeal arch (B) Second pharyngeal arch (C) Third pharyngeal arch (D) Fourth pharyngeal arch (E) Fifth pharyngeal arch"

Second pharyngeal arch The asymmetry in the neck is due to an absence or underdevelopment of the platysma muscle on the patient's right side. The platysma is one of the facial muscles (or muscles of facial expression), even though it is located predominantly in the neck. The defining feature of the facial muscles is their origin or insertion into the subcutaneous tissue, which enables these muscles to convey facial expressions as well as alter the form of the facial orifices (orbits, nostrils, mouth, and external ears). All the facial muscles are derived from the mesoderm of the second pharyngeal (hyoid) arch and are innervated by the facial nerve (CN VII). Additional muscles derived from the second arch include the posterior belly of the digastric, stylohyoid, and stapedius. Choice A (First pharyngeal arch) is incorrect. The first pharyngeal (mandibular) arch folds on itself to form two parts: a dorsal maxillary process and a ventral mandibular process. This complex gives rise to the four muscles of mastication (temporalis, masseter, lateral pterygoid, and medial pterygoid) and four additional muscles: Mylohoid, Anterior belly of the Digastric, Tensor Veli Palatini, and Tensor Tympani (mnemonic = "MATT"). All are supplied by the mandibular division of the trigeminal nerve (CN V3). Choice C (Third pharyngeal arch) is incorrect. The third pharyngeal arch gives rise to only a single skeletal muscle, the stylopharyngeus. This muscle is innervated by the glossopharyngeal nerve (CN IX). Choice D (Fourth pharyngeal arch) is incorrect. The fourth pharyngeal arch is the source of muscles of the soft palate (levator veli palatini, palatopharyngeus, muscle of uvula), pharynx (pharyngeal constrictors, salpingopharyngeus), tongue (palatoglossus), and larynx (cricothyroid). All are supplied by the superior laryngeal branch of the vagus nerve (CN X). Choice E (Fifth pharyngeal arch) is incorrect. The fifth pharyngeal arch is a rudimentary structure, along with the fifth pharyngeal pouch. These pharyngeal arch derivatives regress and disappear early in development. The absence of the fifth arch brings the sixth arch into such a close relationship with the fourth arch, and these pharyngeal arches are often considered a combined 4-6 arch complex. The sixth arch gives rise to most of the intrinsic laryngeal muscles and the esophageal skeletal musculature. The link between the fourth and sixth arches is reinforced by the innervation of the sixth arch by the recurrent laryngeal branch of the vagus nerve. Thus, the vagus nerve supplies both the fourth and sixth arch musculature.

65. A 59-year-old stroke patient is unable to swallow because of a nerve injury. Which of the following nerves is unaffected? (A) Hypoglossal nerve (B) Spinal accessory nerve (C) Vagus nerve (D) Facial nerve (E) Trigeminal nerve

Spinal accessory nerve The spinal accessory nerve supplies the sternocleidomastoid and trape- zius muscles, which are not involved in the act of swallowing. Swallowing involves move- ments of the tongue to push the food into the oropharynx, elevation of the soft palate to close the entrance of the nasopharynx, elevation of the hyoid bone and the larynx to close the opening into the larynx, and contraction of the pharyngeal constrictors to move the food through the pharynx. The hypoglossal nerve supplies all of the tongue muscles except the palatoglossus, which is innervated by the vagus nerve. The vagus nerve inner- vates the muscles of the palate, larynx, and pharynx. The mandibular division of the trigeminal nerve supplies the suprahyoid muscles (e.g., the anterior belly of the digastric and the mylohyoid muscles).

A newborn baby girl is unable to move her head to the right, even when her pediatrician tries to assist the movement, as seen in the photo. Her range of motion in the neck is limited in rotation and lateral bending, and her head posture is abnormally tilted toward the right and her chin is elevated and turned toward the left side. What muscle is most likely responsible for the baby's abnormal range of movement and head posture? (A) Platysma (B) Trapezius (C) Sternocleidomastoid (D) Masseter (E) Digastric"

Sternocleidomastoid The sternocleidomastoid muscle (SCM) is abnormally shortened and/or excessively contracting in this baby girl with congenital torticollis (L: twisted neck). The etiology of congenital torticollis is unknown, but it is thought to be due to damage to the SCM during birth or intrauterine malposition. In this baby, the congenital torticollis presents with the head tilted (or laterally bent) toward the affected SCM (right side in this patient) and the chin is elevated and turned toward the contralateral (left) side. Bilateral contraction of the SCM causes flexion of the neck to move the chin toward the sternum. When the right SCM contracts alone, it functions to bring the mastoid process of the temporal bone closer to the sternum, which results in tilting the head toward the right side and elevation of the chin to the left. The excessive contraction (or tone) of the right SCM causes the inability of this baby to have her head turned to the right side. The accessory nerve (CN XI) provides motor innervation to the trapezius and SCMs, and the congenital torticollis, seen in this patient, is due to shortening or excessive contraction of the right SCM. Choice A (Platysma) is incorrect. The platysma is a muscle of facial expression that resides in the neck and lower face to depress the mandible and wrinkle the skin of neck, as seen when a person is placed in a stressful situation. The platysma originates in the subcutaneous tissue near the clavicle and inserts into the modiolus, lateral to the labial commissures. This muscle is innervated by the cervical branch of the main branch of the facial nerve (CN VII). Due to its superficial origin in the subcutaneous fascia, this muscle can only wrinkle the skin of the neck, not abnormally twist it. This baby has congenital torticollis due to the shortening or excessive contraction of the SCM. Choice B (Trapezius) is incorrect. Damage to the trapezius muscle would lead to asymmetry when shrugging the shoulders or "drooping" of the affected shoulder because the actions of this muscle include elevation and lateral rotation of the scapula during abduction of the upper limb to greater than 90 degrees. The accessory nerve (CN XI) provides motor innervation to the trapezius and SCMs; however, the congenital torticollis, seen in this patient, is due to the shortening or excessive contraction of the SCM. Choice D (Masseter) is incorrect. The masseter muscle primarily works to close the jaw, though its superficial fibers may play a limited role in protrusion of the mandible. The mandibular (third) division of the trigeminal nerve (CN V3) supplies this muscle of mastication, which would have no role in the abnormal twisting of the neck and head posture seen in this newborn. Choice E (Digastric) is incorrect. The digastric muscle is a suprahyoid muscle that attaches to the body and greater horn of the hyoid bone and lies below the body of the mandible. This muscle consists of two muscle bellies, anterior and posterior, which are innervated by the mandibular (third) division of the trigeminal nerve (CN V3) and facial nerve (CN VII), respectively. When it contracts, the digastric muscle acts to elevate the hyoid bone, which is important in swallowing (or deglutition); it is not involved with rotation and lateral bending of the neck

5. A 17-year-old boy receives an injury to the phrenic nerve by a knife wound in the neck. The damaged nerve passes by which of the following structures in the neck? (A) Anterior to the subclavian vein (B) Posterior to the subclavian artery (C) Deep to the brachial plexus (D) Medial to the common carotid artery (E) Superficial to the anterior scalene muscle

Superficial to the anterior scalene muscle The phrenic nerve descends on the superficial surface of the anterior scalene muscle and passes into the thorax posterior to the subclavian vein, anterior to the subclavian artery, and lateral to the common carotid artery. The brachial plexus passes deep to the anterior scalene muscle.

"52. A paralyzed right true vocal fold is most likely associated with which of the following situations? (A) Repair of a patent ductus arteriosus (B) Repair of an aortic aneurysm (C) A gunshot wound below the second rib (D) Surgical removal of the thyroid gland (E) Obstruction of the thoracic duct"

Surgical removal of the thyroid gland The true vocal folds are controlled by the inferior laryngeal nerves, the terminal branches of the recurrent laryngeal nerves off the vagus nerve (CN X). The right recurrent laryngeal nerve branches from the right vagus nerve high in the superior mediastinum, loops under the right subclavian artery, and ascends through the neck along the tracheoesophageal groove. In its course, it runs on the deep aspect of the thyroid gland, where it is vulnerable to injury during thyroidectomy. Lesion of the recurrent laryngeal nerve paralyzes the ipsilateral intrinsic laryngeal muscles (except the cricothyroid) and eliminates sensation below the true vocal fold. Choice A (Repair of a patent ductus arteriosus) is incorrect. The ductus arteriosus is a fetal shunt connecting the root of the left pulmonary artery with the inferior side of the arch of the aorta. It normally closes soon after birth, becoming the ligamentum arteriosum. However, it may remain patent after birth, especially in premature infants, and may require surgical repair. The left recurrent laryngeal nerve branches from the left vagus nerve at the inferior edge of the arch of the aorta, loops under the arch immediately lateral to the ductus arteriosus, and ascends through the neck on a pathway mirroring the right recurrent laryngeal nerve. Because of its close relation to the ductus arteriosus, the left recurrent laryngeal nerve is vulnerable to injury during surgery related to the ductus arteriosus. Choice B (Repair of an aortic aneurysm) is incorrect. The aorta ascends from the heart, curves to the left to form the arch of the aorta, and descends into the thorax offset on the left side of the vertebral column. The left recurrent laryngeal nerve is closely related to the arch of the aorta from its origin to its initial ascent into the neck. It is vulnerable in an aneurysm of the aortic arch, where it may be stretched, and in surgical repair of the aneurysm. Remember that the right recurrent laryngeal nerve is not closely related to the aorta. Choice C (A gunshot wound below the second rib) is incorrect. A penetrating wound directly into the right side of the chest is normally too low to affect the right recurrent laryngeal nerve due to the nerve's high point of origin. However, a similar wound on the left side might affect the left recurrent laryngeal nerve. Choice E (Obstruction of the thoracic duct) is incorrect. The thoracic duct ascends through the thorax, diverges to the left side in the superior mediastinum, and joins the left venous angle (i.e., the junction of the left internal jugular and subclavian veins) in the root of the neck. It is not closely related to the right recurrent laryngeal nerve.

"67. A 40-year-old woman suffers from headaches, nausea, vomiting, and multiple lower cranial nerve involvement. Her physician orders a CT soft tissue neck study, and the given coronal CT shows a mass lesion (tumor) centered at the jugular foramen and identified by arrows. This tumor has destroyed the jugular foramen and hypoglossal canal on the right side and damaged the cranial nerves traversing these foramina. In this patient, which of the following functions will remain intact? (A) Control of the true vocal fold (B) Taste in the anterior two thirds of the tongue (C) Symmetric protrusion of the tongue (D) Sensation in the tympanic cavity (E) Elevation of the shoulder"

Taste in the anterior two thirds of the toungue The chorda tympani nerve, a terminal branch of the facial nerve (CN VII), conveys taste sensation to the anterior two thirds of the tongue and carries presynaptic (preganglionic) parasympathetic fibers to the submandibular and sublingual glands to enable salivation. CN VII enters the skull through the internal acoustic meatus, and in this coronal CT, the margins of this foramen remain intact and are not involved in this presentation. Therefore, the tumor in the posterior cranial fossa is not currently affecting CN VII and its chorda tympani branch, which exits the skull via the petrotympanic fissure. This image shows a massive lesion within the right petrous temporal bone at the right jugular foramen with gross bone destruction. Involvement of the hypoglossal canal was also noted, though not shown on this particular CT. Therefore, the functions of the glossopharyngeal (CN IX), vagus (CN X), accessory nerve (CN XI), and hypoglossal (CN XII) nerves would be lost. Taste supplied to the anterior two thirds of the tongue by the chorda tympani nerve of CN VII would be the only function remaining intact. Choice A (Control of the true vocal fold) is incorrect. All of the intrinsic laryngeal muscles are controlled by branches of the vagus nerve (CN X). Specifically, all of these muscles are supplied by the recurrent laryngeal nerve of CN X, except for the cricothyroid muscle, which is innervated by the external laryngeal nerve off the superior laryngeal nerve of CN X. The pictured lesion has caused gross bone destruction within the right petrous temporal bone at the right jugular foramen. Therefore, the main stem of the vagus nerve is compromised at the right jugular foramen, causing loss of control and sensation in the entire right side of the larynx. Remember that the glossopharyngeal (CN IX), vagus (CN X), and (spinal) accessory (CN XI) nerves emerge from the cranium through the jugular foremen, and the given CT showed erosion of this bony canal by the invading tumor. Choice C (Symmetric protrusion of the tongue) is incorrect. The hypoglossal nerve (CN XII) innervates all of the intrinsic muscles of the tongue and most of its extrinsic muscles with the lone exception being the palatoglossus muscle, innervated by the vagus nerve (CN X). Therefore, damage to the right CN XII would produce ipsilateral deviation of the tongue during protrusion due to the unopposed muscular contractions of the contralateral genioglossus muscle. The mnemonic "The tongue licks the wound" will help you remember that the tongue deviates to the ipsilateral side in a lower motor neuron lesion of CN XII. A patient with a CN XII deficit would also display dysarthria and fasciculations within the tongue musculature, and the right CN XII would have been damaged in this patient due to the invading tumor destroying the hypoglossal canal, as noted in the clinical case. Choice D (Sensation in the tympanic cavity) is incorrect. Sensation from the walls of the tympanic (middle ear) cavity is carried by the branches of the tympanic nerve, a branch of the glossopharyngeal nerve (CN IX). This lesion has caused gross bone destruction at the right jugular foramen, resulting in damage to CN IX. The invading nature of this tumor would cause a sensory deficit in the right tympanic cavity. Remember that the glossopharyngeal (CN IX), vagus (CN X), and accessory (CN XI) nerves emerge from the cranium through the jugular foramen, and this tumor would lead to functional loss of these three cranial nerves. Choice E (Elevation of the shoulder) is incorrect. The (spinal) accessory nerve (CN XI) innervates the trapezius and sternocleidomastoid muscles. The trapezius elevates, retracts, depresses, and rotates the scapula. Thus, damage to CN XI at the jugular foramen results in notable deficits in shoulder action, including elevation. Remember that the glossopharyngeal (CN IX), vagus (CN X), and accessory (CN XI) nerves emerge from the cranium through the jugular foramen, and the given CT showed erosion of this canal by the invading tumor.

1. A 38-year-old man has had thyroid surgery to remove his papillary carcinoma. The external laryngeal nerve that accompanies the superior thyroid artery is damaged during the surgery. This injury could result in a severe impairment of function of which of the following? (A) Relaxing the vocal cords (B) Rotating the arytenoid cartilages (C) Tensing the vocal cords (D) Widening the rima glottidis (E) Abducting the vocal cords

Tensing the vocal chords The external laryngeal nerve innervates the cricothyroid muscle (major tensor), which tenses the vocal cord. The anterior part of the vocalis muscle can tense the vocal cord, and its posterior part can relax the vocal cord. The lateral cricoarytenoid mus- cle rotates the vocal process of the arytenoids cartilage medially, closing the rima glotti- des. The rima glottidis is opened (widened) by rotating the vocal process of the arytenoid cartilage laterally by the posterior cricoarytenoid muscle. Other laryngeal muscles adduct the vocal cords.

"68. The external laryngeal nerve of a 23-year-old man becomes ensnared and tightly compressed by a tortuous superior thyroid artery, which parallels the course of this nerve. Which of the following functions is most likely to be affected? (A) Sensation above the true vocal fold (B) Sensation below the true vocal fold (C) Abduction of the vocal cord (D) Tension of the vocal cord (E) Depression of the hyoid bone"

Tension of the vocal cord The superior thyroid artery accompanies the external laryngeal nerve (or external branch of the superior laryngeal nerve) as they descend from their origins high in the neck. The artery supplies the superior pole of the thyroid gland while the external laryngeal nerve innervates the cricothyroid muscle in the larynx and the lower part of the inferior pharyngeal constrictor. The cricothyroid muscle tilts the thyroid cartilage forward (anterior), causing elongation, tension, and adduction of the vocal cords. Thus, compression of the external laryngeal nerve may reduce the tension of the ipsilateral vocal cord, causing a weakened voice, slight hoarseness, and difficulty raising the pitch of the voice. Choice A (Sensation above the true vocal fold) is incorrect. The true vocal folds (or simply vocal folds) mark the line of separation between the two sensory fields within the larynx. Sensation from the mucosa above the vocal fold (i.e., lining the inlet, vestibule, vestibular folds, and ventricle) is carried by the internal laryngeal nerve, a terminal branch of the superior laryngeal nerve. The external laryngeal nerve is the other terminal branch of the superior laryngeal nerve, and it innervates the cricothyroid and lower part of the inferior pharyngeal constrictor muscles. Choice B (Sensation below the true vocal fold) is incorrect. The (true) vocal folds mark the line of separation between the two sensory fields within the larynx. Sensation from the mucosa below the vocal fold (i.e., lining the infraglottic cavity) is carried by the inferior laryngeal nerve, the terminal branch of the recurrent laryngeal nerve. Choice C (Abduction of the vocal cord) is incorrect. Abduction of the vocal cord (i.e., opening of the rima glottidis) is caused by the contraction of the posterior cricoarytenoid muscle, which laterally rotates the arytenoid cartilage. This muscle is controlled by the inferior laryngeal nerve. Remember that the inferior laryngeal (recurrent laryngeal) nerve supplies all the intrinsic laryngeal muscles except one, the cricothyroid. Choice E (Depression of the hyoid bone) is incorrect. Depression of the hyoid is caused by the actions of the four infrahyoid (strap) muscles (Thyrohyoid, Omohyoid, Sternohyoid, and Sternothyroid). The mnemonic to remember the infrahyoid muscles is "TOSS". Three of the muscles (the "OSS" of the "TOSS") are innervated by the ansa cervicalis, a branch of the cervical plexus. The lone remaining muscle, the thyrohyoid muscle, is innervated by a C1 branch off the hypoglossal nerve (CN XII).

"35. Holoprosencephaly is a complex of developmental abnormalities characterized by the loss of midline structures to greater or lesser degrees. The pictured infant suffers multiple aspects of this disorder, including the midline cleft indicated. This specific developmental malformation is termed "premaxillary agenesis," developmental failure of formation of the intermaxillary segment of the face. Which of the following structures is most likely to be affected in this condition? (A) Nasalbones (B) Softpalate (C) Inferior nasal conchae (D) Philtrum (E) Mandibular incisors"

The answer is D: Philtrum. The intermaxillary segment of the face is derived from the fused medial nasal processes of the facial primordia. It is the midline segment of the upper jaw and is composed of three parts: (1) the philtrum of the upper lip, (2) the premaxilla, and (3) the primary palate. The philtrum is the midline section of the upper lip that normally appears as a small fossa directly under the nasal aperture. The premaxilla is the bony segment that carries the four upper incisors. The primary palate is the small triangular shelf of bone directly posterior to the upper incisors. Its apex is the incisive foramen. Anterior palatal clefts typically lie along one or both lateral edges of the intermaxillary segment, where the medial nasal process would have fused with the maxillary process. Developmental failure of the medial nasal processes to merge causes a midline defect that may include absence of the entire intermaxillary segment (premaxillary agenesis). Choice A (Nasal bones) is incorrect. The nasal bones are derived from the frontonasal process primordial element, perhaps including the medial nasal processes. However, the nasal bones are not components of the intermaxillary segment of the face forming parts of the upper jaw. Choice B (Soft palate) is incorrect. The soft palate is formed from the posterior portion of the secondary palate. The secondary palate is derived from the palatine shelves of the maxillary processes. Posterior palatal clefts are located posterior to the intermaxillary segment and may include defects in the soft palate. Choice C (Inferior nasal conchae) is incorrect. These delicate bones are located on the inferolateral sides of the nasal cavities. Each is derived from the maxillary process, which also gives rise to the maxillary and zygomatic bones in the face and the lateral part of the upper lip, lateral to the philtrum. Choice E (Mandibular incisors) is incorrect. These teeth are located in the lower jaw. However, the upper incisors are held in the premaxillary part of the upper jaw and would be affected by premaxillary agenesis.

"73. Genetic testing of a baby boy with facial and cardiovascular anomalies reveals a small deletion in chromosome 22, specifically 22q11.2. This 22q11.2 deletion (DiGeorge) syndrome often results in migration defects of neural crest cells within the pharyngeal pouches. In this DiGeorge syndrome patient, the thymus and inferior parathyroid glands are absent. Which of the following pharyngeal pouches is most likely affected? (A) First (B) Second (C) Third (D) Fourth (E) Fifth"

Third The thymus and inferior parathyroid glands are derived from the third pharyngeal pouch. Therefore, migration of neural crest cells into the third pharyngeal pouch has been diminished in this patient with DiGeorge (22q11.2 deletion) syndrome. The patient would present with immunodeficiency due to an absence of the thymus, and hypocalcemia (low blood calcium levels) due to the absence of inferior parathyroid glands. DiGeorge syndrome is also characterized by conotruncal hearts defects, such as tetralogy of Fallot or interrupted aortic arch. Moreover, facial anomalies usually resemble first arch syndrome with the patient presenting with a hypoplastic mandible, low-set ears, hypertelorism (increased distance between the eyes), and microstomia (abnormally small mouth). Choice A (First) is incorrect. Evagination of the first pharyngeal pouch forms an elongated diverticulum off the pharynx (the tubotympanic recess). The distal part of this recess widens to form the tympanic (middle ear) cavity. The proximal part remains more tubular and forms the pharyngotympanic (auditory; eustachian) tube. Therefore, agenesis of the thymus and inferior parathyroid glands is not due to maldevelopment of the first pharyngeal pouch. Choice B (Second) is incorrect. The second pharyngeal pouch helps to form the tonsillar fossa and surface epithelium of the palatine tonsil. Secondarily, lymphatic tissue, which becomes the palatine tonsils, is incorporated into the second pharyngeal pouch. Maldevelopment of the second pharyngeal pouch does not cause agenesis of the thymus and inferior parathyroid glands. Choice D (Fourth) is incorrect. The fourth pharyngeal pouch forms the superior parathyroid gland and the ultimobranchial body, which gives rise to the parafollicular (C) cells of the thyroid gland. Maldevelopment of the fourth pharyngeal pouch does not cause agenesis of the thymus and inferior parathyroid glands. Choice E (Fifth) is incorrect. The fifth pharyngeal pouch only exists transiently in human embryologic growth and development. No adult structures are derived from the fifth pharyngeal pouch, so this option can be easily eliminated.

"61. An 8-year-old boy comes to his physician with a painless and smooth mass located in the midline of his neck at the level of the hyoid bone, as noted by the arrow in the given photo. This palpable, midline neck mass was asymptomatic, but due to recent expansion, it has caused difficulty and pain when swallowing. When he swallows or protrudes his tongue, the mass moves superiorly. What is the most likely diagnosis? (A) Enlarged deep cervical lymph node (B) Thyroid nodule (C) Benign parathyroid adenoma (D) Thyroglossal duct cyst (E) Branchial cyst"

Thyroglossal duct cyst A thyroglossal duct cyst is a fibrous cyst that forms within the thyroglossal duct, so it is located in, or close to, the midline of the neck. Most commonly (∼50%), it is found near the body of the hyoid bone; however, the thyroglossal cyst in this patient is located inferior to this point. The thyroglossal cyst is usually painless and smooth, and this mass moves upward during swallowing or protrusion of the tongue, as reported in this patient. The thyroglossal duct is a remnant of the descent of the thyroid gland, which first appears as a single, ventromedian diverticulum (thyroid diverticulum) off the floor of the embryonic pharynx between the tuberculum impar and copula of the incipient tongue. As the thyroid gland descends along the midline, anterior to the gut tube, it remains connected to the tongue by a narrow canal called the thyroglossal duct. The thyroglossal duct usually solidifies and is obliterated after the final descent of the thyroid gland; however, if it persists, a thyroglossal cyst may develop, usually in individuals under the age of 20. Choice A (Enlarged deep cervical lymph node) is incorrect. The deep cervical lymph nodes lie within or in close proximity to the carotid sheath correlating in location with the internal jugular vein. While a few of the deep cervical lymph nodes could reside near the location of the mass, they do not tend to be found superficially on the midline of the neck and an enlarged deep cervical lymph node would not lead to the dysphagia (or difficulty swallowing) seen in this patient. Choice B (Thyroid nodule) is incorrect. A thyroid nodule refers to any abnormal growth that forms within the thyroid gland. Adult women (4% to 8%) are particularly prone to thyroid nodules, but fortunately, only 10% of thyroid nodules are reported to be cancerous. The majority of thyroid nodules are asymptomatic; however, if the cells of the thyroid nodule are producing thyroid hormones, either thyroxine (T4) or triiodothyronine (T3), a thyroid nodule can lead to hyperthyroidism. Patients with hyperthyroidism may present with heart palpitations, weight loss, anxiety, insomnia, fatigue, heat intolerance, excessive sweating, exophthalmos (protruding eyes), and even amenorrhea (an absence of menstrual flow). Due to the midline location of this mass, lack of hyperthyroidism symptoms, and age of this patient, a thyroglossal cyst is the most likely diagnosis. Choice C (Benign parathyroid adenoma) is incorrect. A parathyroid adenoma is a benign tumor of the parathyroid glands, which usually increases the circulation of parathyroid hormone (PTH). This condition of hyperparathyroidism leads to an increase in blood calcium levels due to elevated resorption of bone and may be asymptomatic in many patients. Symptomatic patients would present with lethargy, muscle pain, nausea, constipation, confusion, kidney stones, and even an increased risk of bone fractures due to the increased bone resorption. A physician can perform blood tests to test for calcium, chloride, potassium, and bicarbonate levels, and women over the age of 60 have the highest risk for developing hyperparathyroidism. The presence of a parathyroid adenoma has been reported in 80% to 85% of patients who present with hyperparathyroidism; however, the symptoms and age of this patient do not correlate with the presence of a parathyroid adenoma. Choice E (Branchial cyst) is incorrect. Branchial cysts are located along the anterior border of the sternocleidomastoid muscle. Most often, these cysts are the remnants of the second pharyngeal cleft, located just below the angle of the mandible. Second branchial cleft cysts represent approximately 67% to 93% of all pharyngeal apparatus anomalies. However, branchial cysts may be found anywhere along the anterior margin of the sternocleidomastoid muscle but not on the anterior midline of the neck. Very frequently, branchial cysts are inconspicuous at birth, becoming evident as they enlarge throughout childhood. Due to the midline location of this mass, a branchial cyst can be ruled out in this patient.

41. A 59-year-old man complains of numbness in the anterior cervical triangle. Therefore, damage has occurred to which of the following nerves? (A) Phrenic nerve (B) Greater auricular nerve (C) Transverse cervical nerve (D) Supraclavicular nerve (E) Lesser occipital nerve

Transverse cervical nerve The transverse cervical nerve turns around the posterior border of the sternocleidomastoid and innervates the skin of the anterior cervical triangle. The phrenic nerve, a branch of the cervical plexus, contains motor and sensory fibers but no cutane- ous nerve fibers. The greater auricular nerve innervates the skin behind the auricle and on the parotid gland. The supraclavicular nerve innervates the skin over the clavicle and the shoulder. The lesser occipital nerve innervates the scalp behind the auricle.

"46. If normal evagination of the endodermal lining of the embryonic pharynx does not take place between the pharyngeal (branchial) arches, the pharyngeal (branchial) pouches will not form. Maldevelopment of the first pharyngeal pouch during embryonic weeks 4 to 5 is most likely to result in a congenital disorder of which of the following structures? (A) Thyroid gland (B) Thymus gland (C) Parathyroid glands (D) Facial muscles (E) Tympanic cavity"

Tympanic cavity Evagination of the first pharyngeal pouch forms an elongate diverticulum off the pharynx (the tubotympanic recess). The distal part of this recess widens to form the tympanic (or middle ear) cavity. The proximal part remains more tubular and forms the pharyngotympanic (auditory; eustachian) tube. Development of the tubotympanic recess induces the formation of the overlying first pharyngeal cleft (groove), which forms the external acoustic meatus. The closing plate (pharyngeal membrane) between the tubotympanic recess and the first pharyngeal groove (i.e., the tissue interface between the two) forms the tympanic membrane (eardrum). Choice A (Thyroid gland) is incorrect. The thyroid gland first forms as a single, ventromedian diverticulum (thyroid diverticulum) off the floor of the embryonic pharynx, between the tuberculum impar and copula of the incipient tongue. It descends caudally along the midline, anterior to the gut tube, until achieving its final location in the neck. The thyroid gland is not derived from the pharyngeal pouches, so it can be easily eliminated as an answer. Choice B (Thymus gland) is incorrect. The thymus gland begins as bilateral epithelial primordia in the ventral portions of the third pharyngeal (branchial) pouches, which lose their connections with the pharyngeal walls, descend caudally and medially, and ultimately migrate into the upper anterior thorax where they fuse with each other. Descent of the thymic primordia is notably influenced by shifting of the embryonic heart into the differentiating thorax. Choice C (Parathyroid glands) is incorrect. The parathyroid glands develop from the epithelia of both the third and fourth pharyngeal pouches. The inferior parathyroid appears in the dorsal part of the third pouch, near the incipient thymus gland. The parathyroid is drawn caudally with the migration of the thymic tissue, eventually fixing into place near the caudal dorsal aspect of the thyroid gland. The superior parathyroid first forms in the dorsal part of the fourth pharyngeal pouch. This tissue attaches to the migrating thyroid gland and takes position in the more cranial dorsal aspect of the thyroid. Choice D (Facial muscles) is incorrect. The facial muscles (muscles of facial expression) are small skeletal muscles that form from the mesoderm of the second (hyoid) pharyngeal arch. Other muscles derived from the second arch include the stapedius (in the middle ear), and the stylohyoid and posterior belly of the digastric (in the upper neck). All the second arch muscles are supplied by the facial nerve (CN VII).

13. A 24-year-old man falls from his motorcycle and lands in a creek. Death may result from bilateral severance of which of the following nerves? (A) Trigeminal nerve (B) Facial nerve (C) Vagus nerve (D) Spinal accessory nerve (E) Hypoglossal nerve

Vagus nerve Bilateral severance of the vagus nerve (CN X) causes a loss of reflex con- trol of circulation because of an increase in heart rate and blood pressure; poor digestion results because of decreased gastrointestinal motility and secretion; and difficulty in swal- lowing, speaking, and breathing occurs because of paralysis of laryngeal and pharyngeal muscles. All of these effects may result in death. Bilateral severance of other nerves does not cause death.

"39. A 50-year-old man presents with recurring dizziness, ataxia, vertigo, aphasia, and weakness in his right upper limb. A magnetic resonance angiogram (MRA) revealed a stenosis of the right subclavian artery (marked by the arrow) and poststenotic dilatation, which led to the diagnosis of subclavian steal syndrome. In this condition, blood is shunted from the left side arterial tree via collateral flow into the right side circulation. Through which of the following ipsilateral vessels is blood entering the right subclavian artery distal to the stenosis? (A) Internal thoracic artery (B) Common carotid artery (C) Vertebral artery (D) Superior thyroid artery (E) Suprascapular artery"

Vertebral artery The stenosis of the right subclavian artery depicted in the MRA causes reduced blood flow to the right upper limb, leading to weakness. This stenosis causes the anatomical subclavian steal syndrome, in which the right vertebral artery, which is dilated in the provided MRA, delivers blood back into the occluded subclavian artery. With the stenosis of the proximal end of the right subclavian artery, the resulting arterial pressure differentials enable blood to enter the right vertebral artery after being "stolen" from the left (contralateral) internal carotid tract via the cerebral arterial circle (of Willis) and basilar circulation within the skull. In this patient, blood would travel retrograde within the right vertebral artery to circumvent the stenosis in the proximal right subclavian artery and re-establish blood flow to the right limb. This collateral pathway enables continued use of the right limb; however, the blood being diverted away from the brain can cause brainstem and/or cerebral ischemia, and possibly lead to a stroke. The symptoms seen in this patient, including dizziness, ataxia, vertigo, and aphasia, are indicators of the vertebrobasilar insufficiency, which tells the physician that the collateral circulation is emanating from the cranial circulation. A mnemonic for the four branches of the subclavian artery is "VITamin C," which stands for the Vertebral artery, Internal thoracic artery, Thyrocervical trunk, and Costocervical trunk. Choice A (Internal thoracic artery) is incorrect. The internal thoracic (internal mammary) artery arises from the subclavian artery as its second branch, typically distal to the origin of the vertebral artery. It descends into the thorax along the edge of the sternum to supply the thoracic and abdominal walls, and it receives extensive and important collateral connections along its course. Theoretically, the internal thoracic artery could contribute to bypassing the stenosis of the subclavian artery; however, this artery is not dilated in the provided MRA and its involvement in this case of anatomical subclavian steal syndrome would not account for the cranial ischemia and resulting symptoms. Choice B (Common carotid artery) is incorrect. The brachiocephalic trunk ends by dividing into its terminal branches, the right common carotid and right subclavian arteries. The stenosis depicted in the given MRA is distal to the origin of the common carotid artery, so it would not deter blood from entering into the common carotid artery and would not set up the arterial pressure differentials needed for reverse flow within it. The common carotid artery has no direct connections distal to the stenosis of the subclavian artery, so this artery is unable to bypass the occlusion. Choice D (Superior thyroid artery) is incorrect. The superior thyroid artery is the first branch of the external carotid artery, which supplies the thyroid gland and neighboring muscles. If blood traveled retrograde through the superior thyroid artery, it would travel back toward the common carotid artery, which is located proximal to the stenosis of the right subclavian artery. Choice E (Suprascapular artery) is incorrect. The thyrocervical trunk arises as the third branch of the first part of the subclavian artery, proximal to the anterior scalene muscle. The thyrocervical trunk typically divides into four branches: suprascapular, inferior thyroid, ascending cervical, and transverse cervical arteries. The suprascapular artery runs across the root of the neck to the superior border of the scapula where it supplies blood to the dorsal aspect of the scapula. It does not have any relations to the branching pattern needed to compensate for the stenosis of the subclavian artery in this anatomical subclavian steal syndrome.

51. During a gang fight, a 17-year-old boy is punched, and his nasal septum is broken. Which of the following structures would be damaged? (A) Septal cartilage and nasal bone (B) Inferior concha and vomer (C) Vomer and perpendicular plate of ethmoid (D) Septal cartilage and middle concha (E) Cribriform plate and frontal bone

Vomer and perpendicular plate of ethmoid The nasal septum is formed primarily by the vomer, the perpendicular plate of ethmoid bone, and the septal cartilage. The superior, middle, and inferior conchae form the lateral wall of the nasal cavity. The ethmoid (cribriform plate), nasal, frontal, and sphenoid (body) bones form the roof. The floor is formed by the palatine process of the maxilla and the horizontal plate of the palatine bone.

"24. A 36-year-old woman comes to her physician complaining of heart palpitations, weight loss, anxiety, insomnia, fatigue, and amenorrhea. The physician palpates a 1.5-cm mass on her neck, which elevates when she swallows, located inferior to the cricoid cartilage yet off the midline. What is the most likely structure involved with her presentation? (A) Enlarged deep cervical lymph node (B) Thyroid nodule (C) Benign parathyroid adenoma (D) Thyroglossal duct cyst (E) Branchial cyst"

thyroid nodule A thyroid nodule refers to any abnormal growth that forms within the thyroid gland. Adult women (4% to 8%) are particularly prone to thyroid nodules, but, fortunately, only 10% of thyroid nodules are reported to be cancerous. The majority of thyroid nodules are asymptomatic; however, if the cells of the thyroid nodule are producing thyroid hormones, either thyroxine (T4) or triiodothyronine (T3), a thyroid nodule can lead to hyperthyroidism. Patient with hyperthyroidism may present with heart palpitations, weight loss, anxiety, insomnia, fatigue, heat intolerance, excessive sweating, exophthalmos (protruding eyes), and even amenorrhea (an absence of menstrual flow). Due to the symptoms of this patient and the mass being located at the location of the thyroid gland, a thyroid nodule is most likely diagnosis in this patient. Choice A (Enlarged deep cervical lymph node) is incorrect. The deep cervical lymph nodes lie within or in close proximity to the carotid sheath correlating in location with the internal jugular vein. While a few of the deep cervical lymph nodes could reside near the location of the mass, an enlarged deep cervical lymph node would not lead to the symptoms that brought this patient to the doctor, which were characteristic of hyperthyroidism. Choice C (Benign parathyroid adenoma) is incorrect. A parathyroid adenoma is a benign tumor of the parathyroid glands, which usually increases the circulation of parathyroid hormone (PTH). This condition of hyperparathyroidism leads to an increase in blood calcium levels due to elevated resorption of bone and may be asymptomatic in many patients. Symptomatic patients would present with lethargy, muscle pain, nausea, constipation, confusion, kidney stones, and even an increased risk of bone fractures due to the increased bone resorption. A physician can perform blood tests to test for calcium, chloride, potassium, and bicarbonate levels, and women over the age of 60 have the highest risk for developing hyperparathyroidism. The presence of a parathyroid adenoma has been reported in 80% to 85% of patients who present with hyperparathyroidism; however, the symptoms of this patient do not correlate with the presence of a parathyroid adenoma. Choice D (Thyroglossal duct cyst) is incorrect. The thyroid gland first appears as a single, ventromedian diverticulum (thyroid diverticulum) off the floor of the embryonic pharynx, between the tuberculum impar and copula of the incipient tongue. It descends along the midline, anterior to the gut tube, remaining connected to the tongue by a narrow canal (the thyroglossal duct). The thyroglossal duct usually solidifies and is obliterated after the final descent of the thyroid gland into its normal terminal position. A thyroglossal cyst is a cystic remnant of the thyroglossal duct. It is always located in or close to the midline of the neck. Most commonly (∼50%), it is found near the body of the hyoid bone. Because the mass in this patient is located off the midline, a thyroglossal duct cyst is unlikely. Moreover, a thyroglossal cyst would not cause the symptoms seen in this patient. Choice E (Branchial cyst) is incorrect. Branchial cysts are located along the anterior border of the sternocleidomastoid muscle. Most often, these cysts are the remnants of the second pharyngeal cleft, located just below the angle of the mandible. Second branchial cleft cysts represent approximately 67% to 93% of all pharyngeal apparatus anomalies. However, branchial cysts may be found anywhere along the anterior margin of the sternocleidomastoid muscle. Very frequently, branchial cysts are inconspicuous at birth, becoming evident as they enlarge throughout childhood. Due to the location of the mass, symptoms of the patient, and age of the patient, a branchial cyst can be ruled out in this patient.


Related study sets

AP LANG- Mary Wollstonecraft, "A Vindication of the Rights of Women" Questions

View Set

NUR 214 (Mental Health - Test 3) NCLEX Style Practice Questions

View Set

8.03 Renewable and Nonrenewable Resources

View Set

Intro to Nursing - Exam 3 Concepts - Modules 9, 10, 11, 12

View Set

Ch. 10 Strategy and the Master Budget

View Set